Small animal medicine and metabolic disorders : self-assessment color review [Second edition] 9781138035720, 1138035726, 9781138392434, 113839243X

636 101 66MB

English Pages [287] Year 2019

Report DMCA / Copyright

DOWNLOAD FILE

Polecaj historie

Small animal medicine and metabolic disorders : self-assessment color review [Second edition]
 9781138035720, 1138035726, 9781138392434, 113839243X

Table of contents :
Content: Cardiology. Diagnostic Testing. Effusions. Electrolytes and Acid/Base. Endocrine. Esophageal. Gastric Diseases. Hematology. Hepatobiliary. Immune Mediated. Infectious Diseases. Intestinal/Abdominal Emergencies. Large Intestine. Lower Urinary Tract. Miscellaneous Metabolic Disorders. Neurology. Oncology. Oral Cavity. Pancreas. Rectoanal. Renal Diseases. Reproductive. Respiratory. Small Instestinal.

Citation preview

Self-Assessment Color Review

Small Animal Medicine and Metabolic Disorders 2nd Edition

Self-Assessment Color Review

Small Animal Medicine and Metabolic Disorders 2nd Edition

Edited by

Craig Ruaux

CRC Press Taylor & Francis Group 6000 Broken Sound Parkway NW, Suite 300 Boca Raton, FL 33487-2742 ©  2019 by Taylor & Francis Group, LLC CRC Press is an imprint of Taylor & Francis Group, an Informa business No claim to original U.S. Government works Printed on acid-free paper International Standard Book Number-13: 978-1-138-39243-4 (hardback) 978-1-138-03572-0 (paperback) This book contains information obtained from authentic and highly regarded sources. Reasonable efforts have been made to publish reliable data and information, but the author and publisher cannot assume responsibility for the validity of all materials or the consequences of their use. The authors and publishers have attempted to trace the copyright holders of all material reproduced in this publication and apologize to copyright holders if permission to publish in this form has not been obtained. If any copyright material has not been acknowledged please write and let us know so we may rectify in any future reprint. Except as permitted under U.S. Copyright Law, no part of this book may be reprinted, reproduced, transmitted, or utilized in any form by any electronic, mechanical, or other means, now known or hereafter invented, including photocopying, microfilming, and recording, or in any information storage or retrieval system, without written permission from the publishers. For permission to photocopy or use material electronically from this work, please access www.copyright. com (http://www.copyright.com/) or contact the Copyright Clearance Center, Inc. (CCC), 222 Rosewood Drive, Danvers, MA 01923, 978-750-8400. CCC is a not-for-profit organization that provides licenses and registration for a variety of users. For organizations that have been granted a photocopy license by the CCC, a separate system of payment has been arranged. Trademark Notice:  Product or corporate names may be trademarks or registered trademarks, and are used only for identification and explanation without intent to infringe. Visit the Taylor & Francis Web site at  http://www.taylorandfrancis.com  and the CRC Press Web site at  http://www.crcpress.com 

Preface In the nearly 20 years since the first edition of this book was written, the world of veterinary medicine has undergone remarkable changes. Digital radiography and high-resolution ultrasonography equipment are becoming ubiquitous in companion animal practices, while high-speed computed tomography (CT) and magnetic resonance imaging (MRI) are increasingly common. Together, these advances in imaging technology have dramatically enhanced the ability of veterinarians to identify disease. Along with rapid development in diagnostic imaging, the profession is experiencing an explosive growth in specialized diagnostic tests for many diseases. We now know that pancreatitis in cats is common, yet the way that the disease manifests is very different between dogs and cats. Diagnosis and assessment of respiratory diseases is much more accurate and meaningful with blood gas analysis, CT examination, and bronchoscopy. Some diseases once thought idiopathic are now known to have infectious etiologies. The total amount of knowledge available to clients and practitioners has grown exponentially and continues to do so. All of these changes increase the challenge of modern internal medicine practice but also allow us to treat our patients more effectively. The aim of this book is to present an assortment of cases and case-related materials that are appropriate to the day-to-day practice of small animal medicine, predominantly using example cases that could present well to a generalist veterinary practice. Our hope is that the book will be of value for everyone—from senior students studying internal medicine to specialists in training and experienced internal medicine practitioners desiring a refresher. These cases cover a wide spectrum of metabolic, endocrine, immune-mediated, inflammatory, and infectious diseases. Cases range in difficulty from simple bacterial infections to complex, multisystem disorders that would challenge most practitioners. Some are straightforward in approach, while for others the optimum approach is an area of controversy. The authors are from varying practice backgrounds and specialties, and each brings their own expertise and experience to problem identification and management. Cases are presented in random order and, in many cases, the presentation occurs in several parts as problems are identified, testing is carried out, and then therapeutic plans are made. Cases are grouped by broad subject area in the Broad Classification of Case Numbers listing below. Clinical chemistry values and diagnostic test results are given in both SI and imperial units where possible. As laboratories vary with respect to expected values, reference intervals are provided for each case that are appropriate to the laboratory carrying out the testing.

v

Broad Classification of Case Numbers Cardiology 86, 90 Diagnostic Testing 29, 94, 130, 139 Effusions 16, 107 Electrolytes and Acid/Base 22, 83, 120 Endocrine 17, 21, 25, 30, 33, 39, 57, 62, 71, 76, 80, 82, 96, 104, 109, 110, 113, 123, 142 Esophageal 32, 51, 75, 89, 118, 124 Gastric Diseases 10, 36, 79, 112, 115, 116 Hematology 6, 8, 38, 60, 127, 148 Hepatobiliary 3, 6, 12, 20, 24, 31, 37, 47, 50, 72, 88, 103, 150, 152 Immune Mediated 121, 134, 153, 154 Infectious Diseases 43, 44, 45, 48, 53, 54, 64, 67, 77, 91, 106, 126, 136, 145, 146, 149 Intestinal/Abdominal Emergencies 111, 143, 148

Large Intestine 11, 34, 40, 49, 58, 69, 70, 151 Lower Urinary Tract 85, 95, 97, 108 Miscellaneous Metabolic Disorders 23, 46, 55, 68, 84, 128, 132, 147 Neurology 61, 93, 101, 140 Oncology 70, 105, 152 Oral Cavity 15 Pancreas 27, 35, 41, 117 Rectoanal 19, 99, 125, 131, 144 Renal Diseases 7, 9, 73, 74, 114, 133, 135 Reproductive 18, 78, 81, 98, 138 Respiratory 1, 4, 26, 28, 52, 59, 63, 66, 87, 122, 141 Small Intestinal 2, 13, 14, 42, 56, 65, 92, 100, 102, 119, 129, 137

vii

Contributors Lindsay Gilmour  Texas A&M University College of Veterinary & Biomedical Sciences College Station, Texas Yuri Lawrence  Texas A&M University College of Veterinary & Biomedical Sciences College Station, Texas Jonathan Lidbury  Texas A&M University College of Veterinary & Biomedical Sciences College Station, Texas

Susanne Stieger-Vanegas  Department of Clinical Sciences Oregon State University Corvallis, Oregon Katie Tolbert  Small Animal Clinical Sciences The University of Tennessee Knoxville, Tennessee Austin Viall  Veterinary Pathology College of Veterinary Medicine Iowa State University Ames, Iowa

Craig Ruaux  Massey University School of Veterinary Science Palmerston North, New Zealand

ix

1 & 2: Questions 1  A 6-year-old cat presents with progressively stertorous respiratory sounds, periods of open-mouth breathing at rest, and reduced exercise tolerance. There is no history of sneezing or coughing and no external nasal discharge. Airflow is absent from both external nares. Flexible endoscopic examination of the retropharynx reveals an obstructing soft tissue mass occluding both choanae (Figure  1.1). The obstructing tissue is smooth and light pink in color.

FIGURE 1.1 

i.  What is your clinical diagnosis? What other differential diagnoses should you consider for a mass in this region? ii.  What, if any, additional diagnostic imaging would you recommend in this case? Why? iii.  What treatment options are available for nasopharyngeal polyps? 2  A client presents with a mixed breed dog that has chronic, mixed, small and large intestinal diarrhea. The client is convinced that their dog has a gluten sensitivity and tells you that “ everyone knows that dogs can get celiac disease.”  i.  Gluten-sensitive enteropathy has been conclusively diagnosed in which dog breed(s)? ii.  Which protein(s) do patients with gluten-sensitive enteropathy react to? iii.  Your client is determined that their dog should switch to a gluten-free diet, before any other diagnostic steps. How would you respond?

1

1 & 2: Answers 1  i.  The history, clinical signs, and endoscopic imaging results are most consistent with the presence of an inflammatory polyp occupying the retropharynx and choanae. Differential diagnoses that should be considered in this region include nasal lymphoma, carcinoma/adenocarcinoma, and granuloma formation. ii.  Nasopharyngeal polyps arise from either the Eustachian tubes or middle ear, secondary to chronic inflammation. It is important to assess the middle ears for evidence of disease (otitis media), as this can influence both therapeutic planning and the prognosis for recurrence. Plain radiography can be utilized, but there is the potential for a false negative diagnosis. In some studies, plain radiography has a false negative rate of 25% in cats with otitis media. Advanced imaging, either computed tomography or magnetic resonance imaging, are very useful in assessing the middle ears and status of the osseous bulla. The presence of severe bulla disease argues for bulla osteomy surgery. iii.  If accessible, particularly when the polyp is visible within the caudal oral cavity with retraction of the soft palate, manual distraction of the polyp may be attempted. In the case shown here, the polypoid tissue was not accessible for manual extraction and surgical resection along with bulla osteotomy was carried out. 2  i. Gluten-sensitive enteropathy was definitively diagnosed in the Irish Setter in the United Kingdom in the late part of the 20th  century (Figure  2 .1). The condition had an autosomal recessive mode of inheritance in these dogs. While conclusive diagnosis of gluten-sensitive enteropathy in dogs is now rare, it is reasonable to expect that some individual dogs may present with gluten-responsive gastrointestinal disease. At the time of writing, there is growing controversy in the human medical literature regarding “ non-celiac gluten sensitivity,”  an umbrella term for gastrointestinal symptoms that show response to gluten avoidance yet have not met the typical diagnostic criteria for celiac disease or FIGURE 2.1  wheat allergies in human patients. ii. The major protein(s) leading to gastrointestinal inflammation in patients with gluten-sensitive enteropathies are the gliadins. Ideally, this disease is diagnosed by documenting resolution of clinical signs on a strict gluten-free diet followed by return of clinical signs after an oral gluten challenge. The Irish Setter cases were diagnosed in this manner, leading to a firm diagnosis of gluten-sensitive enteropathy. iii.  As long as the new diet is appropriately nutritionally balanced and appropriate for the dog’ s age and breed, there is no real argument against making this diet change. However, while gluten sensitivity certainly has been well documented in a select group of dogs (the Irish Setters), this does not mean that other diagnostic investigations such as parasitology, assessment of water-soluble vitamins, and diet modification trials with new protein sources can be neglected. Combining diet trials with a gluten-free formulation and a novel protein source would be an ideal way to acknowledge the client’ s concern and if successful would support the diagnosis of food-responsive enteropathy in this patient.

2

3 & 4: Questions 3  A 1-year-old male Pug was presented with a 2-month history of inappetence and mild diarrhea. The appetite is reported to wax and wane. The patient has been tempted with a number of different novel protein diets. The diarrhea is characterized as small intestinal. The patient has not responded to courses of metronidazole, fenbendazole, and symptomatic administration of maropitant. The patient is currently on recommended vaccinations and receives routine heartworm and external parasite prophylaxis. On physical examination, the dog has a normal body temperature (38.6° C , 101.5°  F) and weighs 5  kg (11.0  lb) with a 2/9 body condition score. There was generalized muscle wasting, but otherwise the physical examination was unremarkable. Preliminary diagnostic evaluation demonstrates a mild increase in alanine aminotransferase activity (2× upper limit of normal) and a mild decrease in serum albumin and cholesterol. Other pertinent clinical chemistry results are provided below.

FIGURE 3.1 

FIGURE 3.2 

SI Units

Conventional Units

Result

Ref Range

Result

Ref Range

BUN

2.0

(2.2–5.8)

µ mol/L

10

(13–35)

mg/dL

Glucose

4.5

(3.9–6.9)

mmol/L

90

(70–125)

mg/dL

Analyte

Cholesterol

1.9

(2–4.6)

mmol/L

70

(75–175)

mg/dL

Albumin

25

(26–40)

g/L

2.5

(2.6–4)

g/dL

Total bilirubin

0.5

(0–8.6)

µ mol/L

0.1

(0–0.5)

mg/dL

Alkaline phosphatase

60

(10–70)

U/L

60

(10–70)

U/L

GGT

5

(1–8)

U/L

5

(1–8)

U/L

ALT

170

(5–65)

U/L

170

(5–65)

U/L

3

3 & 4: Questions A CT angiographic study of the portal venous system was performed. Transverse and dorsal plane images are presented (Figures 3.1–3.3). A portosystemic shunt (asterisk, Figures 3.1 and 3.2) arises from the splenic vein and terminates in the caudal vena cava (C) from the left side, just cranial to the celiac artery. Cranial to the insertion of the shunting vessel, the caudal vena cava is widened. Cranial to the insertion of the splenic vein, the portal vein abruptly narrows (arrow, Figure 3.3). The liver is small and only very small portal branches are seen entering hepatic parenchyma (not shown). The patient FIGURE 3.3  has a few small mineral cystoliths (not shown). RK = right kidney, ST = stomach, SP = spleen, LV = liver, A = aorta. i.  What is your interpretation of the imaging findings? ii.  What is the most likely diagnosis? iii.  What additional diagnostic testing could be used to confirm your diagnosis? iv.  How would you approach the management of this case? v.  What is the likely prognosis? 4  A 9-year-old spayed female cat presents with a history of progressive stertor. Figure 4.1 shows the caudal nasopharynx as visualized by flexible endoscopy. i.  What is your clinical diagnosis? ii.  How does this lesion arise? iii.  What treatment options are available?

FIGURE 4.1 

5

3 & 4: Answers 3  i. The patient has a single, extrahepatic portosystemic shunt with secondary microhepatia and cystolithiasis. The splenocaval morphology of the shunting vessel is one of the most common types. ii.  Extrahepatic portosystemic shunt. iii.  Computed tomographic study, bile acids, ammonia tolerance test, scintigraphy. iv.  Medical management (anti-epileptic drug therapy, lactulose, restricted protein diet, and antimicrobial therapy) followed by surgical attenuation of the shunting vessel is recommended. Protein restriction sufficient to ameliorate clinical signs is often attainable with moderate protein restriction (i.e., prescription renal diet, geriatric diets) and is recommended over severe protein restriction in hepatic diets if possible in juvenile patients. v. The prognosis with surgical attenuation is excellent, but a population of dogs that present with neurologic signs may have intractable seizures post-attenuation. Furthermore, seizures as a sole complaint would be an atypical presentation for portosystemic shunts, and these patients may have genetic epilepsy in addition to a portosystemic shunting vessel. 4  i.  There is marked nasopharyngeal stenosis, with almost complete occlusion of the nasopharynx by a membranous soft tissue structure. Compare the image in the question (Figure 4.1) with Figure 1.1 in Question 1.

FIGURE 4.2  Sagittal reconstruction (soft tissue window) of the skull and retropharynx in a cat with nasopharyngeal stenosis. There is marked thickening of the soft palate and retropharygeal mucosa. Cranial to the soft palate, there is substantial fluid and gas accumulation.

ii.  Nasopharyngeal stenosis is an uncommon disorder in cats. Rarely, this condition will occur as a congenital disorder, similar to choanal atresia in human beings. More commonly in cats, the obstructing membrane appears to develop secondary to chronic upper-airway inflammation. iii. The obstructing membrane can occur in varying thicknesses, from 1–2 mm to >2 cm. Cats with thin membranes can often be treated via surgical resection of the inflammatory membrane following an approach via the soft palate. This cat has a longer obstructing structure (Figure 4.2), where complete surgical resection is less feasible. For cases such as this, serial balloon dilation or the use of balloon-expandable metallic stents should be considered.

6

5: Questions 5  An 11-year-old female spayed terrier cross dog presents with an enlarged abdomen (Figure 5.1). On palpation, there is an enlarged liver; no other significant findings are noted, and TPR is unremarkable.

FIGURE 5.1 

i. Using the DAMNIT system, generate a list of differential diagnoses for hepatomegaly in this case. ii.  What would be the highest yield diagnostic procedure for your next step?

7

5: Answers 5  i.  The acronym DAMNIT includes a large number of broad etiological categories and is useful for the generation of an initial list of potential diagnoses that can then be used in making diagnostic plans. Not all etiological categories will apply to all problems. Potential etiologies for hepatomegaly using this system include: Degenerative/Developmental – No relevant diagnoses Anomalous/Autoimmune – Cysts (may be associated with polycystic kidney disease) Metabolic/Mechanical – Vacuolar hepatopathies – Glycogen accumulation (i.e., steroid hepatopathy, diabetes mellitus) – Fat accumulation – Venous congestion (mechanical), (i.e., right-sided heart failure) – Nodular hyperplasia Neoplastic/Nutritional – Primary neoplasia – Hepatocellular carcinoma – Hemangiosarcoma – Biliary carcinoma – Secondary/Systemic neoplasia – Lymphosarcoma – Mast cell tumor – Metastasis (i.e., Hemangiosarcoma) Infect​ious/ ​Idiop​athic​/ Infl​ammat​ory/I​schem​ic/Ia​troge​nic – Parasitic cysts (i.e., Echinococcus) – Liver lobe torsion (Ischemic, congestion) – Granulomas Toxic/Traumatic – Hematoma ii.  If available, high-resolution abdominal ultrasonography would be the highest yield diagnostic step at this point. This will allow observation of the character (diffuse vs. discrete) of the hepatic enlargement and provides an avenue for minimally invasive sampling of many lesions using fine-needle aspiration. In this specific patient, abdominal ultrasonography showed a diffusely enlarged hyperechoic liver consistent with steroid hepatopathy and bilateral marked adrenomegaly consistent with pituitary-dependent hyperadrenocorticism.

8

6: Question 6  A 9-year-old male castrated domestic shorthair cat is examined for the complaint of anorexia. The patient was previously diagnosed with diabetes mellitus 2 years prior. The cat is quiet but responsive on examination. He is tachycardic (230 BPM) but with normal respiratory rate and temperature. Moderate hepatomegaly is noted on abdominal palpation, along with a full, distended bladder. His coat is dry and unkempt. The remainder of the physical examination is unremarkable. Serum chemistry analysis found the cat was markedly hyperglycemic (glucose = 452 mg/dL (25 mmol/L), reference interval: 80–130 mg/dL (4.4–7.2 mmol/L)), the cat also had glucosuria and ketonuria. Complete blood count (CBC) data and a blood smear photomicrograph from the cat (Figure 6.1) are provided.

FIGURE 6.1 

9

6: Question Complete Blood Count Hematology (Units)

Hematocrit (%) Hemoglobin (g/dL) RBC (x106/μL)

Reference Interval

Patient

Flag

24.4

L

30–45

7.9

L

8.0–15.0

5.73

L

5.0–10.0

MCV (fL)

42.5

39.0–45.0

MCH (pg)

13.7

12.5–17.5

MCHC (g/dL)

32.3

30.0–36.0

RDW (%)

18.1

15.0–22.0

Reticulocyte percentage (%)

1.5%

Absolute reticulocyte count (x103/μL)

88.0

WBC (x 103/μL)

14.73

H

5.5–19.5

13.3

H

2.5–12.5

• Segmented neutrophils (x 103/μL) 3

• Band neutrophils (x 10 /μL)

0.1

• Lymphocytes (x 10 /μL)

0.7

• Monocytes (x 103/μL)

0.4

3

0.0–0.3 L

1.5–7.0 0.0–0.85

• Eosinophils (x 10 /μL)

0.1

0.1–0.8

• Basophils (x 103/μL)

0.0

0.0–0.1

3

Platelets (x 103/μL)

251

L

300–800

MPV (fL)

20.3

H

8.6–18.9

RBC, WBC, and platelet morphology:

RBC: See blood smear photomicrograph WBC: Mild neutrophil toxic changes (increased cytoplasmic basophilia and cytoplasmic foaminess) Platelet: Few platelet clumps

i.  What erythrocyte morphologic abnormalities are evident in the blood smear? ii.  Which ancillary stain can be used to highlight these morphologic abnormalities? iii.  What is the significance of these morphologic changes? iv. Are there other erythrocyte cytomorphologic changes that indicate oxidative damage? v.  What compounds or metabolic diseases can cause oxidative erythrocyte damage in the cat and dog?

11

6: Answers 6  i. The majority of the erythrocytes contain Heinz bodies. These structures are consolidations of oxidatively damaged hemoglobin which often are directly connected to the plasma membrane. Cytologically, Heinz bodies are eosinophilic, round projections which typically arise along the edge of erythrocytes (Figure 6.2, black arrows).

FIGURE 6.2 

Low numbers of Ghost cells are also identified on the blood smear. Ghost cells are the remnant plasma membranes of erythrocytes that have lysed intravascularly. Since these membranes no longer contain hemoglobin, Ghost cells have a very lightly eosinophilic to colorless coloration. Ghost cells can occur as a sequel to Heinz body formation. ii.  Due to their eosinophilic coloration, which is nearly identical to the hue of erythrocytes, Heinz bodies can be difficult to identify with the Romanowsky stains (e.g., DiffQuik) used commonly in clinical practice. However, application of new methylene blue stain can highlight Heinz bodies and ease their detection. Heinz bodies appear as deeply basophilic, round inclusions with a new methylene blue stain (Figure 6.3). To perform a new methylene blue stain, mix 0.5% new methylene blue solution with EDTA whole blood at a 1:1 ratio, incubate at ambient temperature for 15 minutes and then prepare a blood smear. iii.  Heinz bodies indicate erythrocytes have been exposed to a strong oxidant; these oxidants can either be endogenously generated compounds or come from an exogenous source. In the cat, up to 5% of circulating erythrocytes may normally contain Heinz bodies. The low-level presence of Heinz bodies in the cat may result from feline hemoglobin containing eight highly reactive sulfhydryl groups relative to just four less reactive sulfhydryl groups present in canine hemoglobin. These reactive sulfhydryl groups make feline hemoglobin relatively more susceptible to oxidative damage from normal physiologic processes and very prone to oxidative denaturation in the presence of a strong oxidant.

12

6: Answers

FIGURE 6.3 

Oxidative insult to erythrocytes can lead to anemia through a number of mechanisms. Ghost cells may occur secondary to Heinz body formation, since membrane-associated Heinz bodies weaken membrane integrity and increase rigidity. These changes prevent erythrocyte deformation when flowing through arterioles or capillaries, which may result in erythrocyte lysis. Oxidation can convert hemoglobin iron from a functional ferrous form (Fe2+) to nonfunctional ferric form (Fe3+), producing methemoglobin, which cannot carry molecular oxygen. Cumulatively, these pathologic changes can result in either an intravascular and/or extravascular hemolytic anemia, commonly termed “Heinz body anemia.” In this particular case, the cat’s Heinz body anemia is suspected to be secondary to his diabetic ketoacidosis. iv.  In addition to Heinz bodies and Ghost cells, oxidative erythrocyte insult can produce eccentrocytes and schistocytes. Eccentrocytes form when two opposing sides of the plasma membranes are oxidatively fused together. This membrane fusion displaces the RBC hemoglobin to the other side of the cell, creating a crescent-shaped clearing zone in the erythrocytes. Schistocyte erythrocyte fragments may result from intravascular lysis of the damaged erythrocytes. Free Heinz bodies can also be identified in some instances. Blood smear evidence of erythrocyte regeneration, such as increased numbers of polychromatophils and nucleated erythrocytes, may also be observed. v.  Erythrocyte oxidative insult may result from exposure to a spectrum of exogenous toxins, endogenously generated compounds, and metabolic diseases. In the cat, Heinz body formation has been reported to occur secondary to administration or ingestion of acetaminophen, propylene glycol, propofol, phenazopyridine, benzocaine, methylene blue, and Allium plant products (e.g., garlic powder). Heinz body formation can also be observed in both cats and dogs with chronic renal insufficiency and diabetic ketoacidosis. In these cases, accumulation of plasma uremic acids and ketoacids, respectively, may be partially responsible for the oxidative insult. Heinz bodies and eccentrocytes have also been documented in dogs and cats with lymphoma and cats with hyperthyroidism.

13

7 & 8: Questions 7  You are assessing a 10-year-old neutered male Border Collie for polyuria and polydipsia. On a routine chemistry panel, the laboratory reports a value for SDMA of 17  µ g /dL (Reference Range: 0– 14  µ g /dL). The dog’ s BUN and creatinine are both within the laboratory’ s reference ranges but are in the higher part of the reference ranges. i.  What does the acronym SDMA refer to? ii.  How do you interpret these findings? iii.  How would you plan to further assess this dog? 8  The following table shows hemograms from two dogs. Parameter

Dog A

Dog B

RBC (x10 /L)

3.6

1.9

5.5–8.5

Hb (g/L)

95

2.4

120–180

12

Hct (%)

Reference Interval

22

15

37–55

11.9

22.5

11.6–14.8

MCV (fL)

52

79.3

62–77

MCHC (g/L)

280

295

320–360

RDW (%)

i.  Which of these hemograms is more consistent with regeneration following blood loss and which is compatible with iron deficiency anemia? ii.  What is a more specific indication of regeneration than the results given above? iii.  How do cats differ from dogs when it comes to identification of regeneration?

15

7 & 8: Answers 7  i.  SDMA is an acronym for Symmetric DiMethylArginine, a metabolic by-product of methylation of arginine residues in proteins. SDMA shows almost exclusively renal excretion, with no tubular resorption. SDMA appears to be less susceptible to changes due to muscle mass, age, and disease stage than creatinine. ii.  This value represents a moderate increase in SDMA above the laboratory reference range. With the history of polyuria/polydipsia and high normal BUN and creatinine, a clinical suspicion of renal insufficiency is warranted. iii.  A complete urinalysis is warranted given this dog’ s history and clinical signs; this would remain the case even if SDMA were within the reference range. Given the elevation in SDMA documented here and compatible clinical signs, IRIS staging of the dog’ s renal disease is warranted. Additional tests necessary at this stage to complete staging of the dog include blood pressure measurement, urinary sediment exam, and, if the sediment is inactive, urine protein to creatinine ratio (UP:C). 8  i.  The hemogram from Dog B is consistent with regeneration following blood loss. Note that the red cell distribution width (RDW) is increased, indicating greater than normal variability in red cell sizes, while the MCV is mildly increased, indicating that on average the red cells are larger than usual. Together the increased RDW and MCV indicate the presence of larger, typically younger red blood cells, as we expect with red cell regeneration. The hemogram from Dog A shows a microcytic, hypochromic (low MCV and MCHC, respectively) anemia with no evidence of regeneration (RCV is normal). This pattern of microcytic hypochromic anemia is characteristic of iron deficiency anemia. ii. Reticulocytosis (i.e., increased circulating reticulocytes) is a more specific and definitive indication of red cell regeneration. iii. In the dog, all reticulocytes are reasonably considered immature to young red blood cells. In the cat, there are two distinct forms of reticulocyte that may be seen, aggregate and punctate. Only the aggregate reticulocyte is taken as evidence of regeneration, as punctate reticulocytes can circulate for >7 days and thus are not accurate indicators of recent red cell release into the circulation.

16

9 & 10: Questions 9  You are assessing a patient with a vague complaint of lethargy and “ not doing well.”  On routine biochemical examination, you note that the patient has azotemia. i.  What are the three broad categories of azotemia? List some conditions or diseases that may lead to each type. ii.  How would you go about distinguishing between these types of azotemia? What additional diagnostic testing should you consider? 10  Figure 10.1 presents an endoscopic view of the greater curvature of a dog’s stomach.

FIGURE 10.1 

i.  What is the structure visible against the gastric wall? ii.  What are the indications for using this device? iii.  What are the pros and cons of this device in comparison to naso-esophageal (NE) feeding tube use?

17

9: Answers 9  i.  The three broad categories of azotemia are Pre-Renal, Renal, and Post-Renal. Examples of conditions or diseases that may lead to each of these are given in the table below. Pre-Renal Azotemia

Conditions resulting in reduced renal blood flow: Dehydration Hypovolemia Hypotension Congestive heart failure Hypoadrenocorticism Renal vascular thrombosis

Renal Azotemia

Primary kidney diseases: Acute kidney injury Chronic renal insufficiency Glomerulonephritis Pyleonephritis

Post-Renal Azotemia

A. Urinary tract obstruction: Ureterolithiasis Urethrolithiasis Ureteral or urethral neoplasia Accidental ligation of ureters B. Urinary tract rupture: Blunt abdominal trauma Bladder laceration (cystocentesis accident) Secondary to urinary tract obstruction Ureteral avulsion (abdominal trauma)

ii.  Urine specific gravity, serum BUN/Urea:Creatinine, abdominal fluid analysis, and diagnostic imaging, particularly abdominal ultrasonography, may all assist in categorizing the type of azotemia present. Response to therapy may also be informative. With pre-renal azotemia, there is typically sufficient functional renal mass to allow concentration to occur. Patients with pre-renal azotemia are usually oliguric and what urine is produced is adequately to dramatically concentrated, with urine specific gravity >1.035 in cats and >1.040 in dogs. The BUN:Creatinine is typically elevated in patients with pre-renal azotemia, due to differences in handling of these 2 substances. Pre-renal azotemia usually resolves rapidly with fluid therapy if identified and treated in a timely manner. Patients with renal azotemia may be oliguric, anuric or polyuric. If a patient is dehydrated and azotemic, and urine specific gravity falls between 1.007–1.029 (dogs) or 1.007– 1.039 (cats), the most likely cause is intra-renal disease. With progression of renal insufficiency, the kidney may lose both concentrating and diluting function, resulting in persistent isosthenuria (urine specific gravity 1.008–1.012). Urine specific gravity with post-renal azotemia is highly variable and thus this condition cannot be immediately diagnosed on the basis of a spot urine specific gravity. Typically, this diagnosis will be made from a combination of history (often trauma), presence of abdominal effusion with elevated creatinine and diagnostic imaging (see Question 97).

18

10: Answers 10  i.  This is a percutaneous endoscopic gastrostomy, or PEG, tube. ii.  These tubes are particularly useful for the management of severe esophageal disease, such as ulceration, strictures, and severe esophageal mucosal trauma. PEG tubes may also be used post-surgically in animals that have sustained severe oral trauma or have marked abnormalities in esophageal motility such as megaesophagus or esophageal diverticulum. While PEG tubes can aid in nutritional support for patients with esophageal dysmotility, they do not reduce the risk of aspiration pneumonia in these patients. iii.  In comparison to NE feeding tubes, PEG tubes have substantially larger internal diameters, which broadens the potential diets that can be used. Blenderized, canned food with a small amount of water are feasible with PEG tubes, while diets used with NE tubes will typically need to be purpose-made liquid diets. PEG tubes can remain in place for an extended period (months or more), while long-term maintenance of NE tubes is essentially impossible. PEG tubes allow bolus meal feeding, making home feeding more feasible for many owners. PEG tubes are, however, more expensive and placement is both more invasive and requires greater technical skill and equipment support than for an NE tube. PEG tube failure, while rare, can result in accidental introduction of food into the abdominal cavity and development of life-threatening peritonitis.

19

11: Questions 11  Figures 11.1 and 11.2 show fluorescence in situ  hybridization (FISH)-stained biopsy samples from the colon of a dog with chronic hematochezia and diarrhea. The FISH stain is using a eubacterial DNA probe.

FIGURE 11.1 

FIGURE 11.2 

i.  What do the FISH-stained biopsies show? ii.  What breed(s) of dog are typically affected by this disease? iii.  What, if any, treatment options are available?

21

11: Answers 11  i.  The FISH stain is highlighting the presence of bacteria (the red/orange structures indicated in Figure  11.3) intracellularly within the mucosa. In normal dogs, bacteria are only present on the surface of the mucosa (Figure  11.4). FISH staining uses fluorescent probes that interact with specific DNA sequences; the eubacterial probe interacts with DNA with all bacterial types and species. The host cell nuclei are stained with a differing blue fluorescent stain.

FIGURES 11.3 and 11.4  FISH staining of a canine colon sample shows intracellular bacteria (asterisk) deep within the crypts of the colonic mucosa. Compare to the normal canine colon in Figure  11.4 (Right), where the bacteria are present only within the lumen of the colonic crypts, and none are present intracellularly.

ii.  This FISH result is characteristic of granulomatous colitis of Boxers. As the name suggests, the Boxer is the breed most commonly affected, but the same pathology has been reported in a limited number of French Bulldogs. iii.  The specific organism invading the mucosa in these dogs is a strain of Escherichia coli . This organism is sensitive to antibiotic therapy, but the antibiotic chosen needs to be able to reach effective concentrations intracellularly. Empirically, enrofloxacin has been used successfully in many dogs. Unfortunately, fluoroquinolone resistance is now emerging in these cases, and thus culture of the organism and determination of antimicrobial sensitivity is recommended.

22

12: Questions 12  A 7-year-old female spayed English Bull terrier presents with an overnight history of acute vomiting and lethargy. She is up-to-date on all vaccinations. The dog has vomited at least four times overnight and is anorexic. On physical examination, she is icteric. Her owner reports that she has had previous bouts of gastroenteritis after digging in their garden and scavenging garbage. Pertinent results from clinical biochemistry are provided. SI Units Analyte

Result

Conventional Units

Ref Range

Result

Ref Range

BUN/Urea

9

(2.5–9)

µ mol/L

25.2

(7–25.2)

mg/dL

Creatinine

110

(48–109)

µ mol/L

1.2

(0.5–1.2)

mg/dL

Cholesterol

10.8

(3.27–9.82)

mmol/L

415.4

(125.8–377.7)

mg/dL

Total protein

49

(54–74)

g/L

4.9

(5.4–7.4)

g/dL

Albumin

31

(33–44)

g/L

3.1

(3.3–4.4)

g/dL

1.1

(0.1–0.2)

mg/dL

Total bilirubin

18

(1–3)

µ mol/L

Creatine kinase

467

(0–385)

U/L

Alk. phos

1944

(0–87)

U/L

ALT

9378

(0–88)

U/L

AST

11973

(0–51)

Na+

151

(141–153)

U/L mmol/L (mEq/dL)

K+

3.8

(4–5.4)

Calcium

2.6

(2.08–2.82)

mmol/L

mmol/L (mEq/dL) 10.42

(8.34–11.3)

mg/dL

Phosphorus

1.25

(0.92–1.82)

mmol/L

3.87

(2.85–5.63)

mg/dL

i.  How do you interpret the clinical chemistry results provided? What are some differential diagnoses you are considering? ii.  What is the serum half-life of alanine transaminase in the dog and how does this differ from the cat? iii.  What are the three basic mechanisms for the development of icterus? Which do you think is most important in this case?

23

12: Answers 12  i.  Dramatic elevations in AST, ALT, and ALP activity and bilirubin concentration are noted. There are also mild elevations in CK activity and cholesterol concentration, while total protein and albumin are reduced. These results are consistent with a severe, acute hepatopathy, either inflammatory or necro-inflammatory. While all of ALP, AST, and ALT are altered, the relative change is greater in the cellular leakage enzymes (ALT, AST) than the cholestatic marker ALP, implying that hepatocellular necrosis and inflammation are the primary insult. Cholestasis is more likely to be secondary to the liver insult in this case. The mild decreases in total protein and albumin may indicate the onset of hepatic insufficiency (loss of albumin synthetic capacity), but albumin is also a negative acute phase protein in the dog, tending to show decline in the face of severe inflammatory disease as hepatic synthetic activity is diverted to other acute phase proteins. AST activity is the most sensitive of the liver enzyme activity for acute hepatocellular injury but is less specific than ALT, as muscle also contains large amounts of AST, and thus acute muscle injury, can result in elevated AST activity. In this case, however, there is only a mild elevation in CK activity while AST is enormously elevated, and other markers of hepatic injury are altered (ALT, bilirubin), making muscle injury less likely as a cause of the AST elevation. Differential diagnoses for acute liver injury in dogs include infectious causes (bacterial, viral), hepatic necrosis (thrombosis), toxin exposures, severe abdominal trauma, and use of hepatotoxic drugs. Given the history of this dog, toxin exposures and infectious (bacterial) causes are most likely. The dog has no history of drug exposure or abdominal trauma. The dog is vaccinated against canine adenovirus type-1. Given the prior history of gastrointestinal signs following garbage scavenging, exposure to toxins in moldy garden waste or spoiled food is the most likely diagnosis, but acute bacterial hepatitis cannot be ruled out at this stage and empirical treatment for bacterial hepatitis is appropriate. ii.  The serum half-life of ALT in the dog is around 60  hours (2.5  days), while in cats ALT’ s half-life is substantially shorter (6 weeks, variable to poor appetite and weight loss. Routine clinical chemistry, CBC, and fecal parasitology examinations are unremarkable, and physical examination is unrewarding. The owner has limited resources and does not wish to proceed with advanced diagnostics such as ultrasonography or endoscopy with intestinal biopsies without good reason. How would you address this case?

25

13: Answer 13  Chronic enteropathy is a clinical diagnosis but does not indicate any particular etiology. When owner finances are limited, as is often the case, it is important to be able to convincingly justify any further interventions. In this scenario, the potential diagnostic and therapeutic yield of further steps is crucially important. Diagnostic imaging has very little likelihood of yielding a diagnosis that will alter therapeutic plans in a dog with these specific findings. If a mass lesion or alteration in intestinal character were palpable, the diagnostic yield of imaging (particularly abdominal ultrasonography) becomes much higher, and this modality is easier to justify. Potential causes of chronic enteropathic signs in dogs are numerous, but many of the potential etiologies are relatively uncommon. At this stage, parasitism is reasonably ruled out. Important differential diagnoses for this dog, in no particular order, are: • • • •

Food-responsive enteropathy (FRE) Antibiotic-responsive diarrhea (ARD) Idiopathic inflammatory bowel disease (IBD) Exocrine pancreatic insufficiency (EPI)

Other differential diagnoses such as intestinal neoplasia and other diseases associated with chronic weight loss, are much less likely given the clinical findings so far. From a diagnostic yield perspective, it is relatively easy to justify the measurement of trypsin-like immunoreactivity (TLI), cobalamin, and folic acid in this patient. A normal TLI concentration would rule out exocrine pancreatic insufficiency as a source of clinical signs. Subnormal cobalamin would also be an important finding, as this is known to be a negative prognostic factor for response to therapy in all of the remaining differential diagnoses and the need for supplementation can be established with this testing. Food-responsive enteropathy is an important differential in these cases that is often overlooked. Assuming measurement of TLI rules out EPI, a dietary trial with a hypoallergenic or novel protein source is indicated. It is important to emphasize to the owner that this diet trial must be strictly followed. Ideally, the trial should last for at least 4 weeks, but in many cases substantial improvement is seen within 14 days in patients with FRE. Antibiotic therapy trials should not be started until after a food trial has been conducted and has failed. Many dogs with FRE will show some response to antibiotic therapy simply due to the antibiotic’ s effect on the intestinal microbiome, but it is preferable to avoid unnecessary antibiotic therapy. If a food trial fails, trial therapy with antibiotics (typically, tylosin is recommended) for a period of several weeks is a rational next step. Rapid resolution of clinical signs within days of beginning therapy would support the diagnosis of ARD. This also implies that antibiotic therapy will be an important factor in the management of this dog in the long term. Animals failing food and antibiotic trials are highly likely to have IBD, with other differential diagnoses much less likely at this stage. At this point, if there is no prospect for further diagnostic investigations, it is reasonable and appropriate to trial therapy with anti-inflammatory doses of glucocorticoid steroids.

26

14 & 15: Questions 14  A 6-year-old female spayed Weimaraner (Figure 14.1) was given a clinical diagnosis of inflammatory bowel disease because of chronic small intestinal diarrhea, mild weight loss, and variable, typically poor, appetite. There were no other diagnostic tests performed. She has received 2 mg/kg of oral prednisolone daily for the last 6 weeks. She is presenting to you for a second opinion due to ongoing, dramatic weight loss.

FIGURE 14.1 

i.  What possibilities are you considering to explain the ongoing weight loss? ii.  What are some common side effects of high-dose glucocorticoid therapy? iii.  How would you plan to approach this case? 15  A Border terrier dog presents to your clinic with a complaint of ptyalism and loss of appetite. These signs had been noticed by the dog’s owner upon picking it up from a boarding facility. On physical examination, the dog resents palpation of the face and muzzle. The dog is anesthetized for a more complete oral examination (Figure 15.1).

FIGURE 15.1 

i.  What differential diagnoses are you considering for the etiology of these lesions? ii.  If this dog had been electrocuted, what other clinical signs and changes would you expect to see? iii.  How would you plan to support/treat this dog?

27

14 & 15: Answers 14  i.  Given that the diagnosis is based purely on history and presenting complaint, there is a very real chance that this patient has been misdiagnosed. Other diseases associated with marked weight loss, such as neoplasia (particularly small intestinal, given the history of diarrhea), end-stage renal disease, protein-losing enteropathies, and nephropathies should be considered. This dog has also been chronically receiving high doses of glucocorticoid steroids and muscle mass loss is a recognized side effect of chronic high doses of glucocorticoids. ii.  Along with muscle mass loss, common side effects of chronic high doses of glucocorticoids in dogs include polyuria/polydipsia, polyphagia, tachypnea, sleep disturbances, hepatomegaly, thinning of the skin, development of a pendulous abdomen (due to both hepatomegaly and loss of tone in the abdominal rectus muscles), and elevation in serum ALP activity. iii. It is critical to obtain an accurate diagnosis in this patient. At a minimum, screening, biochemical testing, fecal parasite screening, and a full urinalysis should be obtained. Diagnostic imaging is indicated. The highest yield imaging modalities for use in this case would be abdominal ultrasonography or computed tomography examination and thoracic radiography. Measurement of the serum concentrations of B-group vitamins, particularly cobalamin (vitamin B12) and folic acid/folate (vitamin B9), should be carried out and supplementation should be provided if deficiencies are noted. Cobalamin deficiency in particular is associated with a poor response to therapy for chronic enteropathies and is a poor prognostic finding. Gastrointestinal biopsies are necessary if no other diagnosis is obtained with the previously mentioned steps. Before biopsy, however, the dose of glucocorticoid medications needs reduction to minimize the risk of wound-healing complications. A gradual tapering of glucocorticoid dosing is important to avoid iatrogenic hypoadrenocorticism. Assuming a final diagnosis of idiopathic inflammatory bowel disease in this patient, other therapeutic options should be explored in order to allow reduction in glucocorticoid dosing. Cyclosporine, chlorambucil, and azathioprine are possible second-line therapies. 15  i.  The dog shows significant lingual and oral mucosal ulceration. While not an exhaustive list, potential causes of this ulceration include exposure to corrosive substances, uremic stomatitis, autoimmune diseases such as systemic lupus erythematosus, bullous pemphigoid, and toxic epidermal necrolysis; other potential causes are local thermal damage, electrocution, and Candida infection. ii.  Animals presenting following electrocution (i.e., from chewing on electric cords) typically also show altered mentation and dyspnea associated with non-cardiogenic pulmonary edema and the lesions typically are more linear than the patchy distribution seen here. iii.  Clinical chemistry panels being unremarkable, uremic stomatitis was ruled out. Close questioning of the boarding facility staff revealed that the dog had been exposed to an improperly diluted cleaning agent, leading to corrosive-induced stomatitis. The dog was managed using a combination of narcotic pain control medications, local bathing of the lesions with an oral chlorhexidine product and placement of an esophagostomy tube for nutritional support.

28

16: Questions 16  A 6-year-old castrated male Maine Coon cat presents with severe respiratory distress. Two-view thoracic radiographs are acquired and reveal significant pleural effusion. Thoracentesis removes approximately 250  m L of fluid (Figure 16.1). On analysis, the fluid had a total nucleated cell count of 3,600 cells/µ L and refractometer-measured total protein of 4.9  g /dL. The cytologic features of the fluid are presented in the second image (Figure 16. 2).

FIGURE 16.1 

FIGURE 16.2 

i.  How would you characterize this effusion? ii.  Considering the gross and cytologic features of the fluid, what specific type of effusion is present? iii. What biochemical analyses could be performed to confirm the presence of this effusion? iv.  In the cat, which disease processes are most commonly associated with the development of this effusion type?

29

16: Answers 16  i.  The three principal effusion categories are the pure transudate, modified transudate, and exudate. A fluid is classified as a pure transudate if the total nucleated cell count (TNCC) is  5,000 cells/µ L and TP >  25   g /L. Some effusions will fall outside this classification scheme or are variable in presentation. For example, the TNCC and TP of chyloabdomen or uroabdomen could fall into either the modified transudate or exudate categories, depending upon the case. Some effusions, such as hemoabdomen or hemothorax, are not categorized by this classification system but are readily identified from appearance and cellularity. In this particular case, the TNCC and TP of the cat’ s fluid is consistent with a modified transudate. ii.  Grossly, the fluid is milky white and pink in appearance. Cytologically, lymphoid cells predominate. Most of the lymphoid cells are small lymphocytes with a lesser numbers of intermediate-sized lymphocytes also found. The gross and cytologic features of the fluid are most consistent with a chylous effusion (chylothorax). iii.  Chylous cavitary effusions result from leakage of chyle from lymphatic structures of the chest, abdomen, or mesentery. Chyle is comprised of both lymphatic fluid and dietary-derived lipid components coming from the mesentery. The presence of a chylous effusion can be confirmed by comparing the concentrations of triglycerides in the fluid and serum. If the effusion triglyceride concentration is at least two times greater than the serum concentration of triglycerides, the fluid is considered a chylous effusion. The abundant lipids within chylous effusions artifactually increase the total protein of the fluid if measured with a refractometer. As a result, the measured TP of chylous effusions can vary widely depending upon the lipid content, ranging from 3– 7  mg/dL, if measured via refractometer. iv.  Chylothorax represents up to 30% of feline cavitary effusions. Chylous effusions in cats most commonly occur in association with conditions which increase venous hydrostatic pressures, especially in the cranial vena cava or associated draining greater veins. The primary differential diagnosis to consider for chylothorax in the cat is congestive heart failure secondary to hypertrophic cardiomyopathy. Additionally, cranial mediastinal neoplasia (e.g., thymoma, lymphoma, or thyroid tumor), pericardial effusion, diaphragmatic herniations, or heartworm disease can cause increased venous hydrostatic pressure and subsequent chylous effusion. Chylothorax in the cat has also been reported to occur secondary to thoracic trauma and intrathoracic surgery or can be idiopathic. In this particular case, echocardiography demonstrated hypertrophic cardiomyopathy, a common cardiac disorder of the Maine Coon breed.

30

17, 18, & 19: Questions 17  This ultrasound image (Figure 17.1) shows the midcervical region of a dog, just to the right of midline. i.  Which hormone would you expect to be abnormal in this dog? ii. What electrolyte abnormality would you expect to see in this dog? iii.  What are the most common clinical signs of this endocrinopathy?

FIGURE 17.1 

18 A 6-year-old intact female Labrador retriever dog presents with a soft tissue mass protruding from the vulva (Figure  18.1). i.  What is your clinical diagnosis? ii.  What stage of the estrus cycle is this dog most likely in? iii.  What complications of this condition may occur? iv.  The owners do not wish to breed this dog. What would your therapeutic recommendation be? FIGURE 18.1 

19  A 6-week-old Boston terrier puppy presents for vaccination. The owners report that they have never seen the puppy defecate and that the mother appears to lick the puppy more than others in the litter. Figure 19.1 shows the anal, perineal, and vulval anatomy of this puppy. i. What are the four different categories of atresia ani? ii.  Which, if any, breeds are predisposed to atresia ani? iii. What other anatomical abnormality is most likely present in this puppy?

FIGURE 19.1 

31

17, 18, & 19: Answers 17  i.  Figure 17.1 shows a discrete, hypoechoic nodule within the right thyroid gland, consistent with a parathyroid adenoma. We would expect to see abnormalities in parathyroid hormone concentration. ii.  Parathyroid hormone (PTH) is a major regulator of calcium homeostasis. PTH promotes mobilization from bone and decreases renal calcium excretion; thus the expected abnormality in this dog is hypercalcemia. This specific patient had a total calcium of 3.32 mmol/L (13.3 mg/dL), with the laboratory having an upper reference value of 3 mmol/L (9.3 mg/dL). Ionized calcium was 1.63 mmol/L (1.25–1.45 mmol/L). iii. The most common clinical signs of hyperparathyroidism, in descending order, are polyuria and polydipsia, lethargy, weakness, hyporexia, and lower urinary tract signs related to calcium-oxalate urolithiasis. Essentially, all of these abnormalities are attributable to the presence of persistent hypercalcemia. 18  i. The dog is presenting with vaginal hyperplasia. The vaginal floor has become enlarged and protrudes through the vulva under the influence of estrogen. There is a small number of differential diagnoses for this condition. It is possible for vaginal wall tumors (leiomyoma, leiomyosarcoma) to present in this manner, but this is uncommon. Uterine prolapse may appear similar to this lesion, but prolapse of the uterus typically occurs postpartum. ii.  The dog is most likely in proestrus or estrus. This condition occurs as a result of estrogen-mediated hyperplasia of the vaginal floor mucosa. The condition will resolve rapidly as the dog proceeds into diestrus. iii.  In breeding dogs, copulation and intromission may be hampered. Urinary obstruction due to urethral compression can occur, but is uncommon. If present, urinary obstruction should be managed by catheterization. If breeding is desired for the current estrus, artificial insemination may be necessary. iv. Surgical sterilization (spaying) will rapidly resolve the immediate condition and prevents recurrence via removal of gonadal estrogen. 19  i. There are four different categories of anatomical abnormality that result in atresia ani, types I–IV. Type I atresia ani refers to a congential distal rectal/anal stricture; however, there is a patent lumen remaining and the anus is not imperforate. Both type II and type III atresia ani feature an imperforate anus and blind rectal pouch. In atresia ani type II, the blind rectal pouch is 1.5 cm. The puppy illustrated here has atresia ani type II. The fourth category, atresia ani type IV, is rare and features normal distal rectal and anal anatomy which, however, fails to communicate with a more distant blind rectal pouch. ii.  The Boston terrier and Miniature/Toy Poodle breeds are apparently predisposed, with other breeds sporadically recognized. Females also appear to be overrepresented. This is an uncommon diagnosis. iii.  Given that the puppy does not show abdominal distention or failure to thrive, feces must be able to leave the large intestine in some manner. It is most likely that the puppy has either a recto-vaginal or recto-vestibular fistula, allowing passage of feces via the vagina. This is a common additional abnormality in female dogs with atresia ani, and may explain why females are overrepresented, as recto-vaginal fistulae provide a mechanism for passage of feces that is not available in a male puppy.

32

20: Questions 20  An elderly dog has a long history of elevated ALT activity. The dog presents to you with a complaint of behavioral changes, including circling, aimless wandering, and random vocalizations. Suspecting onset of hepatic encephalopathy you submit serum for a clinical chemistry panel; results are provided. SI Units Reference Range

Conventional Reference Range

SI Result

Low

High

Low

High

BUN

2.2

3.6

11.4

µ mol/L

6.2

10.1

31.9

mg/dL

Creatinine

88

53

123

µ mol/L

1

0.6

1.4

mg/dL

Glucose

2.8

3.9

6.9

mmol/L

50.9

70

125.5

mg/dL

2

3

9

mmol/L

76.9

115.4

346.2

mg/dL

Analyte

Cholesterol Total protein

60

52

75

g/L

6

5.2

7.5

g/dL

Albumin

22

26

44

g/L

2.2

2.6

4.4

g/dL

8

0

6

µ mol/L

0.5

0

0.4

mg/dL

469

0

609

U/L

469

0

609

U/L

Alk. phos

9

0

185

U/L

9

0

185

U/L

ALT

80

0

75

U/L

80

0

75

U/L

Total bilirubin CK

i.  Which parameters in the clinical chemistry panel are reasonable surrogate markers for hepatic function? Why are they markers of hepatic function? ii.  How do you account for the relatively unremarkable hepatic enzyme activity, given this dog’ s history? iii.  What additional diagnostic steps would you take to confirm your clinical diagnosis of hepatic encephalopathy?

33

20: Answers 20  i.  Blood glucose, albumin, cholesterol, and BUN concentrations are all reasonable surrogate markers for hepatic function. These compounds are all synthesized by hepatocytes. Reduction in functional hepatic mass thus leads to decreasing concentrations of these compounds. While these are reasonable markers to support a clinical suspicion of hepatic failure, none of these findings, alone or in combination, are specific for hepatic failure. ii.  Alanine transaminase (ALT) is an intracellular enzyme that is released via “ leakage”  from hepatocytes that have been compromised. Elevations in ALT activity are consistent with the presence of inflammatory or necro-inflammatory diseases of the liver. As hepatic disease progresses, loss of hepatocyte cell mass means that there is a markedly reduced amount of enzyme available to leak, which can bring the serum activity of leakage enzymes close to or within the reference range. iii.  Consider diagnostic imaging (ultrasonography) and liver biopsy for a more conclusive diagnosis of underlying hepatic disease. Specific hepatic function testing can be carried out using measurement of basal ammonia concentration, ammonia tolerance testing, or the use of the pre- and post-prandial bile acids test. Resting bile acid concentrations alone are nonspecific, as gastrointestinal diseases and intestinal dysbiosis can be associated with high resting bile acid concentrations. Ammonia testing is arguably more accurate, but the highly labile nature of ammonia in blood samples makes accurate performance of these tests more difficult.

34

21 & 22: Questions 21 Chubby, a 12-year-old spayed female mixed breed dog is presented to your clinic for progressive hair loss, weight gain, and lethargy. Chubby’  s only pertinent medical history is that she has a complicated urinary tract infection that has been treated with trimethoprim sulfa antibiotic for the past 4 months. On physical exam, she has a dull hair coat with non- FIGURE 21.1  pruritic bilateral alopecia and hyperpigmentation of her tail. Her body condition score is 8 out of 9. Pertinent blood and chemistry results are below. Analyte

Result

Reference Range

Hematocrit

35

Absolute reticulocyte count

10

41–60%

Total protein

6.8

5.4–6.8 g/dL

Cholesterol

450

130–354 mg/dL

12.5–93 ´ 103 /µ L

i.  What is the next most appropriate diagnostic step? ii.  What is the sensitivity and specificity of the recommended diagnostic tests for diagnosis of the dog’ s likely condition? iii.  What is the most likely reason for this dog’ s disease? What therapeutic recommendations would you make? 22  A 10-year-old spayed female dog presents markedly obtunded and weak. Results from a venous blood gas analysis are given below. Analyte

Result

Reference Interval

Chloride

113.0 mmol/L

109–122

Potassium

4.9 mmol/L

3.5–5.8

Sodium

152 mmol/L

144–160

7.2

7.31–7.42

tCO2

14.9 mmol/L

21–31

HCO3–

13.7 mmol/L

20–29

31 mmol/L

17–27

pH

Anion gap

i.  How do you interpret these blood gas/electrolyte values? ii. What are common causes of high anion gap metabolic acidosis in veterinary patients? iii.  This dog has diabetic ketoacidosis and, after appropriate volume replacement with crystalloid fluids, you begin therapy with an intravenous CRI of insulin and dextrose. Which of the electrolytes listed above should you monitor closely, and why?

35

21 & 22: Answers 21  i.  In light of the history and physical exam findings of lethargy, non-pruritic hair loss, and weight gain, and the nonregenerative anemia and hypercholesterolemia, baseline thyroid testing, including fT4 and cTSH should be performed. ii.  The sensitivity and specificity of fT4 and cTSH are approximately 74% and 98%, respectively. iii.  Sulfa drugs are one of the few classes of drugs that can induce reversible hypothyroidism. The dog does not appear to have any predisposing factors for the development of hypothyroidism (e.g., predisposed breed, middle age). Thus, it is highly likely that TMS is the cause of this dog’ s hypothyroidism. Transient treatment of hypothyroidism with l-thyroxine is recommended until TMS can be withdrawn and clinical signs of hypothyroidism resolve. 22  i.  The dog is acidemic (pH 7.2) with reduced bicarbonate/tCO2 , indicating the presence of a metabolic acidosis. The anion gap is elevated, giving a diagnosis of high anion gap metabolic acidosis. ii.  The high anion gap indicates the presence in circulation of acid compounds that do not have chloride as an anion (hence the anion gap; there is not sufficient chloride anion measured to maintain overall electroneutrality). Typically, a high anion gap is due to the presence of organic acids in circulation. Classically, the mnemonic MUDPILES has been used for substances that lead to elevated anion gap in both human and veterinary patients:

M Methanol U Uremia D Diabetic ketoacidosis (although ketoacidosis can also result from starvation) P Paracetamol/acetaminophen I Iron/inborn errors of metabolism L Lactic acidosis E Ethylene glycol or ethanol S Salicyclates

Several of these causes are highly unlikely in companion animals, leaving uremia, diabetic ketoacidosis, lactic acidosis, and ethylene glycol intoxication as the most common causes of high anion gap metabolic acidosis in veterinary patients. iii.  Of the electrolytes listed, potassium is the most likely to undergo significant and potentially life-threatening alterations in patients with diabetic ketoacidosis receiving insulin therapy. Rapid declines in potassium result from the increasing activity of insulin-dependent Na+/K+-ATPase pumps; these pumps translocate potassium from the extra- to intracellular space, potentially resulting in severe hypokalemia.

36

23 & 24: Questions 23  You have been treating a dog for chronic renal insufficiency/failure for a period of several months. The dog presents as an emergency, having had several seizures. On physical examination, the dog is markedly obtunded and has vestibular signs. You measure the dog’ s blood pressure (oscillometric) and determine that the blood pressure is 205/145  m mHg. i.  What are the criteria for a hypertensive crisis? ii.  How would you plan to treat this dog? iii.  How would your treatment plans differ if there was no evidence of target organ damage? 24  You are managing a dog with elevated liver enzyme activity, most notably with an ALT activity of 345 IU/L (Reference Range:  0–80). A liver biopsy shows accumulation of copper in the hepatocytes (Figure 24.1).

FIGURE 24.1  Histological appearance of copper staining in a hepatic biopsy from a dog with a copper-accumulating hepatopathy. Quantitative copper value in this dog was 8,264 PPM. (Image courtesy of Dr. Bob Sherding.)

i.  Which dog breeds are most commonly affected with hepatic copper accumulation disorders? ii.  How is hepatic copper accumulation documented? iii.  How is hepatic copper accumulation treated?

37

23 & 24: Answers 23  i.  Animals with severe hypertension (≥ 180/120  m mHg) and target organ damage are considered to be in hypertensive crisis. End‑organ damage may manifest as CNS signs (as seen here), acute retinal detachment and/or hyphema, acute worsening of congestive heart failure, or rapid progression of renal insufficiency/failure. ii.  Treatment of patients in a hypertensive crisis is aimed at reducing blood pressure within a period of several hours. In dogs, therapy is initiated with either oral or injectable ACE-inhibitor and calcium channel blocker medications. If necessary, direct vasodilators such as hydralazine, nitroprusside, or topical nitroglycerin may be used to rapidly reduce blood pressure. iii.  If no end‑organ damage is apparent when an elevated blood pressure is initially detected, blood pressure should be remeasured after 1 week to document persistence. If the severe hypertension persists, antihypertensive therapy should begin. In dogs, the most common first-line class of medications used is an ACE-inhibitor. Blood pressure should be rechecked after approximately 1 week and, if the systolic blood pressure continues to be ≥ 150/95  m mHg, a second agent, typically a calcium channel blocker, should be added. The aim of therapy is to reduce blood pressure to 50% eosinophils; no infectious agents are seen. ii.  What is eosinophilic bronchopneumopathy? iii. What, if any, breeds are predisposed to this condition? iv.  What underlying disorders may be FIGURE 26.2  associated with eosinophilic bronchial infiltration? v.  How would you plan to treat this dog?

41

26: Answers 26  i.  There is a severe, diffuse bronchointerstitial pattern affecting all lung lobes. In addition, bronchiectasis is visible particularly in the left cranial lung lobes (most readily visible in the right lateral view, Figure  26.1). These findings are consistent with a long-standing infiltrative (most likely inflammatory) disorder of the bronchi and bronchioles. ii.  Eosinophilic bronchopneumopathy, also sometimes referred to as pulmonary infiltrates with eosinophils (PIE), is a severe inflammatory disease of the pulmonary parenchyma and airways, most commonly diagnosed in younger dogs. It usually presents with clinical signs of dyspnea, harsh productive coughing, and reduced exercise tolerance, some dogs will also have mucopurulent nasal discharge or will productively cough a green-yellow mucoid material. iii.  Siberian husky and Alaskan malamute are overrepresented in some published case series. iv.  Eosinophilic airway inflammation may be associated with a variety of infectious aetiologies, including some bacterial and fungal (particularly mold) infections of the airways. In endemic areas, parasites such as heart worm and lung worm (Oslerus osleri , Angiostrongylus vasorum ) should be screened for and treated appropriately. In areas of high lung worm endemism an empirical therapeutic trial with anthelmintic medications, such as fendbendazole, is warranted. Eosinophilic bronchopneumopathy is idiopathic and thus is a diagnosis of exclusion. It is believed to result from an inappropriately severe response to inhaled antigens, but in most cases the inciting allergen is not able to be indentified. v.  Most dogs with eosinophilic bronchopneumopathy respond well to oral glucocorticoid therapy. In some cases glucocorticoid side effects become limiting, and inhaled steroid therapy may become necessary. The bronchiectatic changes are more likely to be permanent, and may require the use of mucolytics and airway humidification to aid in the clearance of mucus.

42

27 & 28: Questions 27 An 11-month-old male neutered Retriever cross dog (Figure 27.1) presents with a complaint of ravenous appetite; diarrhea with large volumes of greasy, fetid stool; and poor weight gain. Results of serum trypsin-like immunoreactivity (TLI), cobalamin and folate assays are given below. i.  What is your clinical diagnosis? ii.  What is the most likely primary disease process underlying this syndrome? iii. How do you interpret the cobalamin and folate results? Do these influence your initial treatment plans? iv. Would you expect this dog to be at greater risk for the development of diabetes mellitus?

FIGURE 27.1 

Test

Patient Result

Reference Interval

TLI

607  nmol/L (22  µ g /dL) at 1 hour post IV administration of ACTH (5  µ g /kg, to a maximum of 250 µ g) are consistent with hyperadrenocorticism. In the case presented here, substantial post-ACTH elevation in cortisol is expected. The dog’  s actual results were 193   nmol/L (6.99   µ  g /dL) pre- and 1231   nmol/L (44.6  µ g /dL) at 1 hour post-ACTH. iii.  The UCCR has high sensitivity (up to 100% in some studies) but low specificity (as low as 20% in some studies) due to the effect of both non-adrenal illness and/or hospitalization stress on cortisol secretion. This test is effective for screening purposes but does not conclusively diagnose hyperadrenocorticism in the absence of other testing. The sensitivity of the LDDS for diagnosis of hyperadrenocorticism (the diagnosis is based ONLY on assessing the 8  hours post-dexamethasone sample) ranges from 80– 85%, with a specificity of 70– 75%. While less nonspecific than the UCCR, this test is still susceptible to false positive results in animals with non-adrenal illness. The greatest utility of this test lies in the potential to distinguish adrenal-dependent from pituitary-dependent disease. With the common availability of abdominal ultrasonography and CT examination of the abdomen, the clinical usefulness of the LDDS has been substantially reduced. The ACTH stimulation test, in cases of pituitary-dependent disease, has a sensitivity of approximately 85% with a specificity of 85–90%.

126

77 & 78: Questions 77  A 2-year-old Labrador retri­ever was seen to be consuming raw salmon meat approximately 5 days prior to presentation at your clinic. The dog has developed acute, severe watery diarrhea, significant pyrexia, and marked enlargement of all peripherally palpable lymph nodes. Cytological preparations of the lymph nodes are shown (Figure 77. 1).

FIGURE 77.1 

i.  What is the causative agent of salmon poisoning disease? ii.  Name the trematode worm that acts as a vector for the salmon poisoning disease agent. iii.  This disease has a highly localized area of occurrence. Where do most cases of this disease arise? 78  A 3-year-old intact female mixed breed dog is examined for the complaint of serosanguineous vaginal discharge. On digital examination of the vaginal vestibule, a round firm mass is palpated on the ventral floor. Small tissue biopsy segments of the vestibular mass are acquired vaginoscopically. An impression smear of the tissue is presented.

FIGURE 78.1 

i.  How would you interpret the cytologic features of the vaginal mass? ii.  What other round cell malignancies can have a similar cytologic appearance? iii. How can you diagnostically differentiate this tumor from other round cell malignancies?

127

77 & 78: Answers 77  i. Salmon poisoning disease is caused by the opportunist infection of dogs with the rickettsial organism Neorickettsia helminthoeca . Observation of this organism, either intracellularly or free within aspirates of affected dog lymph nodes (Figure  77.  2 , arrows), is strongly supportive of this diagnosis. The exuberant lymphocytic proliferative response to this organism can be so marked that it is occasionally mistaken for lymphosarcoma. ii. Transmission of Neorickettsia FIGURE 77.2  helminthoeca  to affected dogs occurs via ingestion of a trematode worm, Nanophytes salmincola , which parasitizes salmonid fish. Consumption of raw fish, or exposure to discarded offal from salmonid fish, is the predominant mode of infection in dogs. iii.  Salmon poisoning disease is most commonly diagnosed in the Pacific Northwest region of the United States and Canada, this being the region with the highest prevalence of N. salmincola . Sporadic cases have been reported in dogs outside this region, almost invariably in association with consumption of raw salmon imported from the Pacific Northwest. This disease is rapidly progressive and has up to 90% mortality if untreated. Thankfully, the disease rapidly responds to tetracycline treatment in most cases, if recognized. 78  i.  A population of atypical round cells is apparent. These cells are found individually and exhibit moderate-marked anisocytosis. The cells have moderate N:C ratio and contain round, peripherally placed nuclei. The nuclear chromatin is granular and many cells contain 1– 3 nucleoli. Their associated cytoplasm is moderately basophilic, with most cells containing punctate clear vacuoles. The cytologic features of this particular tumor are characteristic for transmissible venereal tumor (TVT). Multiple punctate clear vacuoles are found in nearly all cases of TVT and rarely in other round cell malignancies, with the exception of lymphoma. Mitotic figures and multinucleated tumor cells are commonly observed on TVT samples. ii.  Lymphoma can be very difficult to cytologically differentiate from TVT. While the vaginal canal is not a typical anatomic location for canine lymphoma, epitheliotropic T-cell lymphoma may arise at any epithelial lining. Mast cell tumors may arise in this anatomical location but are distinguishable from TVT due to numerous metachromatic granules. iii. Empirical diagnosis of TVT is made from the anatomic location of the lesion, cytologic features of the tumor, and patient history. Definitive differentiation of TVT from lymphoma or plasmacytoma requires histologic evaluation of the mass, possibly coupled with immunohistochemical marker staining.

128

79: Questions 79  You are working up a dog for a chronic enteropathy characterized by weight loss, small intestinal diarrhea, and variable appetite. You obtained endoscopic biopsy samples of the small intestine and gastric mucosa. In the histopathology report, the presence of a moderate population of gastric spiral organisms (GSOs) is reported in the stomach (Figure 79.1).

FIGURE 79.1  H&E (A) and Warthin-Starry (B) stained samples of gastric mucosa from a dog, showing a moderate population of gastric spiral organisms (arrows in A, darkly stained spirals in B) within the gastric mucus layer.

i.  Name some GSO species that are found in dogs and cats. Which, if any, of these are pathogens in human beings? ii.  How common is GSO carriage in normal dogs and cats? iii.  Do you think treatment of the GSOs is justified given this case description?

129

79: Answers 79 i. GSOs commonly identified in companion animals include Helicobacter bizzozeronii, H. felis, H. salomonis, and H. heilmannii. In addition, H. pylori carriage has been established in a feline colony experimentally, but natural carriage is rare. Of the species listed, only H. pylori has been identified as a pathogen in human beings. ii.  GSO carriage is ubiquitous in normal dogs and cats, with a reported prevalence greater than 90% across several large studies. iii.  The role of GSOs as a potential gastrointestinal pathogen is unclear, but on balance it does not appear that they are important in the vast majority of patients. One exception may be if a heavy GSO burden is present in patients with predominantly gastric symptoms (chronic vomiting) and routine therapeutic approaches are failing. In the case described here, there is no sound justification for treatment of the GSOs.

130

80: Questions 80  A 9-year-old neutered male Border Collie cross dog presents with a history of general malaise and weakness which has developed over a period of approximately 1 week and a history of vomiting that morning while out on a walk. The patient has been receiving chronic chlorambucil and prednisolone therapy for previously diagnosed, indolent, marginal zone lymphoma; the prednisolone therapy was withdrawn 1 month previously. On physical examination, the dog is slightly dull but responsive; temperature, pulse, and respiratory rate are all considered normal. Lymph nodes are palpably enlarged but stable in size from recheck examination 1 month prior. The dog readily eats food when offered. Pertinent results from a routine biochemistry panel are given below. SI Units Analyte

Result

“ Conventional”  Units

Ref Range

Result

Ref Range

4.9

(2.5–8)

µ mol/L

13.7

(7–22.3)

mg/dL

114.9

(44.2–159.1)

µ mol/L

1.3

(0.5–1.8)

mg/dL

Glucose

2.2

(3.9–7.9)

mmol/L

40

(70.9–144.5)

mg/dL

Cholesterol

3.1

(2.8–8.3)

mmol/L

118.8

(109.2–317.7)

mg/dL

Total protein

71

(52–82)

g/L

7.1

(5.2–8.2)

mg/dL

BUN Creatinine

Albumin

31

(22–39)

g/L

3.1

(2.2–3.9)

g/dL

Total bilirubin

1.7

(0–15.4)

µ mol/L

0.1

(0–0.9)

g/dL

Creatine kinase

0

(0–0)

U/L

0

(0–0)

U/L

Alk. phos

133

(23–212)

U/L

133

(23–212)

U/L

3

(0–11)

U/L

3

(0–11)

U/L

GGT ALT

90

(10–125)

U/L

90

(10–125)

U/L

Na+

164

(144–160)

mmol/L

164

(144–160)

mEq/dL

K+

4.1

(3.5–5.8)

mmol/L

4.1

(3.5–5.8)

meq/dL

Calcium

2.8

(2–3)

mmol/L

11.2

(7.9–12)

mg/dL

Phosphorus

0.6

(0.8–2.2)

mmol/L

2

(2.5–6.8)

mg/dL

i.  Based on your assessment of the history and biochemistry results, what differential diagnoses are you considering? ii.  What additional diagnostic testing could you use to support your clinical diagnosis? iii.  What diagnostic imaging modalities would you consider in this case?

131

80: Answers 80 i. The only clinically significant abnormality noted on the chemistry panel is hypoglycemia (2.2  m mol/L, 40  mg/dL). Higher likelihood differential diagnoses include insulinoma and hypoadrenocorticism, while less likely diagnoses include hepatic synthetic failure (unlikely, as BUN, cholesterol, and albumin are all within normal), sepsis (unlikely, given unremarkable physical examination findings), iatrogenic insulin overdose (unlikely, given history, the dog does not have a history of diabetes mellitus), and paraneoplastic effects from tumor types other than insulinoma. The dog does have an existing diagnosis of indolent lymphosarcoma, which is occasionally associated with hypoglycemia; however, other tumor types such as hepatocellular carcinoma and leiomyosarcomas are more commonly associated with hypoglycemia. ii.  Hypoadrenocorticism can be most conclusively ruled out with an ACTH stimulation test. If finances are constraining, a resting cortisol within the expected range reduces the likelihood that hypoadrenocorticism is present but cannot completely rule out this disease. With a high index of suspicion for insulinoma, measurement of plasma insulin, and the amended insulin:glucose ratio are indicated. These diagnostic tests are best run when the animal is overtly hypoglycemic; typically it is recommended that these tests should be run when blood glucose is 40 also supports the clinical suspicion of insulinoma. Paraneoplastic hypoglycemia resulting from tumors other than insulinoma results from the production of other hypoglycemia-inducing compounds, such as insulin-like growth factor 1, insulin receptor upregulation, or insulin binding to other proteins being produced by the tumors. As the islet cells are not involved with these conditions, down-regulation of endogenous insulin production occurs, resulting in a low insulin and normal insulin:glucose in these patients. Insulin measurement thus cannot be used to conclusively rule out paraneoplastic hypoglycemia. This patient presents with a relatively short history of clinical signs suggestive of hypoglycemia. Measurement of serum fructosamine concentrations can be used to assess glucose homeostasis over a period of several weeks. iii.  Both abdominal ultrasonography and contrast-enhanced computed tomography of the abdomen (where available) can yield valuable information. The reported sensitivity of abdominal ultrasonography for the detection of the primary tumor in dogs with insulinoma varies from 28–75%, while contrast-enhanced CT angiography has a reported sensitivity of approximately 70% for detection of primary insulinomas within the pancreas, but this is based on a small number of reported patients.

132

81 & 82: Questions 81  Figures  81.1 and 81.2 show the gross appearance and cut section of the uterus removed from a 5-year-old female dog. She has never borne a litter.

FIGURE 81.1 

FIGURE 81.2 

i.  What is cystic endometrial hyperplasia? ii.  Which hormone(s) are responsible for the development of this condition? iii.  What are some risk factors for the development of this condition? iv.  The risk of which disease increases with the presence of CEH? 82  A 10-year-old Miniature Poodle was recently diagnosed with diabetes mellitus and is not yet well regulated based on owner observation of ongoing polyuria/polydipsia and polyphagia. At a recheck examination, the owner comments that the dog’ s “ eyes have gone white”  (Figure  82.1).

FIGURE 82.1 

i.  How common is cataract formation in diabetic dogs? ii.  What mechanism underlies the development of diabetic cataracts? iii.  Will this cataract resolve with better glycemic regulation? What advice will you give the owner?

133

81 & 82: Answers 81 i.  As the name suggests, cystic endometrial hyperplasia is a hyperplastic process affecting the endometrium in reproductively whole female dogs. The prevalence of CEH increases with age of the dog. The lesion is thought to result from a combination of endocrine factors, chronic inflammatory change due exposure to intrauterine bacteria during proestrus and estrus, as well as a reduction in myometrial motility. ii.  Progesterone is the primary hormone responsible for the development of CEH in dogs. This hormone’s effect on the uterine environment includes the stimulation of endometrial gland proliferation and secretion, reduced myometrial motility and closure of the cervix, leading to retention of fluid secretions. Other endocrine factors that may play a role in the development of CEH include exposure to estrogens (which increase endometrial sensitivity to progesterone) and insulin-like growth factor 1 (IGF-1), which also promotes endothelial proliferation. iii.  The primary risk factor for the development of CEH is increasing age in the dog. The female dog experiences a period of prolonged high progesterone in the luteal phase of the estrus cycle, regardless of the presence of pregnancy (i.e., nulliparous individuals will still experience high progesterone, as if pregnant). The effects of progesterone on the uterine environment are cumulative with each estrus cycle. iv.  The presence of CEH is a substantial risk factor for the development of pyometron, bacterial infection of the endometrium. Accumulation of endometrial gland secretions and the reduced uterine motility resulting from progesterone exposure provide an environment that favors the colonization of the uterus by bacteria that can enter the uterus via the open cervix in proestrus and estrus. Exposure to exogenous estrogens, as occasionally used for prevention of pregnancy following mismating, is associated with increased risk of pyometron due to sensitization of the uterus to the effects of endogenous progesterone. 82 i. Cataract formation is the most common chronic complication seen in dogs with diabetes mellitus. Many dogs will already have developed cataracts at the time of diagnosis and up to 50% of dogs will have diabetic cataracts developed within approximately 6 months of initial diagnosis. ii.  Diabetic cataract formation occurs due to osmotic changes within the lens in the presence of hyperglycemia. In normoglycemic dogs, the metabolism of glucose within the lens can proceed normally, but in hyperglycemic dogs, several enzymes important for glucose metabolism can become saturated. This then results in the metabolism of glucose to other end products, particularly sorbitol. The lens capsule is impermeable to sorbitol, resulting in a significant osmotic draw of water into the lens. Excess water within the lens disturbs the orderly arrangement of lens fibril proteins, resulting in a loss of transparency. iii.  Due to the impermeable nature of the lens capsule to sorbitol, the cataracts are irreversible. Thankfully, many dogs are able to adjust to the loss of vision and will continue to be happy, functional pets even in the presence of diabetic cataracts. If the owners wish to attempt a return of vision, cataract removal surgery can be considered and approximately 80% of dogs will show return of vision. The prognosis for a good outcome from cataract surgery is better if the dog is showing good glycemic regulation before surgery. A recent (Williams, DL 2017 Vet Sci. 2017 Mar; 4(1): 18) publication describing a small number of dogs suggests that inhibition of aldose reductase delays the onset of diabetic cataracts. The effect seen was relatively small (~80  days longer duration before cataract development) and the utility of this approach is questionable.

134

83: Questions 83  A 6.5-year-old neutered male domestic longhair cat presents to your clinic severely obtunded, having been missing for approximately 18  hours. The cat was found in lateral recumbency and was minimally responsive. On physical examination, you detect an irregular heart rhythm. A very large, tense urinary bladder is palpable. An ECG trace from this cat is reproduced in Figure  83.1.

FIGURE 83.1 

i.  What changes do you see in the ECG strip? What is the most likely cause of these changes? ii. Other than urinary tract obstruction, what other disorders are associated with hyperkalemia in the cat? iii.  In some patients hyperkalemia is associated with bradycardia. How does hyperkalemia cause bradycardia? iv.  What emergency management steps would you take with this cat?

135

83: Answers 83  i.  The cat appears to be in atrial standstill; there are no P waves visible. A sinoventricular rhythm is present, with marked widening of the QRS complexes and tall, prolonged T waves observed. These changes are most consistent with marked hyperkalemia, at the time of initial presentation this cat’ s serum K+ concentration was 9.7  m mol/L (this value is the same in mEq/L units). ii.  The great majority of causes of hyperkalemia in the cat are related to reduced or absent renal or urinary tract function. Examples include acute oliguric renal failure, ruptured urinary bladder, and urethral obstruction. Insulin deficiency, particularly diabetic ketoacidosis, and reperfusion injury of muscle following aortic thromboembolism can also cause substantial changes in ECF potassium concentration. iii.  The ratio of intra- to extra-cellular potassium concentrations is a critical determinant of the cellular membrane potential. Increased extra-cellular potassium concentrations decrease the resting cell membrane potential and reduce the difference between the resting potential and the threshold for the production of an action potential, so with lower degrees of hyperkalemia there may actually be an increase in the heart rate. However, repolarization of the pacemaker cells after an action potential relies primarily on the efflux of potassium down an intra- to extra-cellular gradient. A marked increase in extra-cellular potassium concentration decreases this gradient, which slows the repolarization of electrically active cells such as the pacemaker cells in the sinoatrial node. iv.  Calcium gluconate infusion can be immediately lifesaving, as it increases cell membrane potential and thus reduces likelihood of ventricular fibrillation. This does not, however, reduce ECF potassium concentration. Administration of dextrose IV or insulin+dextrose in patients with diabetic ketoacidosis promotes translocation of potassium ions into the intracellular space. Crystalloid fluid therapy, using potassium-free or low-potassium fluids (0.9% NaCl, lactated Ringer’ s solution, respectively) can aid in reduction of ECF potassium via dilution. Figure 83.2 shows a repeat ECG from this cat following 7  hours of fluid therapy as described above.

FIGURE 83.2 

Relief of the urethral obstruction is central to the management of cats presenting with this disorder.

136

84 & 85: Questions 84  A 3.5-year-old female spayed Shar Pei dog presents with an owner complaint of swollen hocks (Figure 84.1). On physical examination, the dog is febrile (39.9° C , 103.8°  F). Routine biochemistry, hematology, and urinalysis (via dipstick) are unremarkable.

   FIGURE 84.1 

i.  What is your clinical diagnosis? ii.  What is the underlying pathology that leads to Shar Pei fever? iii.  Which other organ systems are most commonly affected in Shar Pei fever? iv.  What is the mode of inheritance of Shar Pei fever? v.  Name some other species and breeds that are predisposed to renal amyloidosis. 85  A canine patient presents with a complaint of dysuria, pollakiuria, and poor appetite of 2 days duration. A urine sample is obtained by cystocentesis under ultrasound guidance. Figure  85.1 shows a high-power microscopic view of the urinary sediment, stained with a Romanowskytype stain (i.e., Dif-Quik). i.  What do you see? ii.  What are the most common infecting organisms in dogs with urinary tract infections? iii. Urinary tract infections are commonly recognized in dogs newly diag- FIGURE 85.1  nosed with ketoacidosis. What are other common risk factors for the development of diabetic ketoacidosis in dogs?

137

84 & 85: Answers 84  i.  Shar Pei fever, also known as Shar Pei swollen hock syndrome. ii.  Shar Pei fever is a familial amyloidosis, featuring amyloid deposition into the skin and other target organs. iii. Shar Pei fever is most commonly associated with renal (glomerular) amyloid deposition, although other organs, including the liver, pancreas, spleen, myocardium, adrenals, lungs, and CNS may be affected. Extra-renal amyloid deposition is more common in Shar Pei (about 73% of cases) than in non-Shar Pei breed dogs (29%) with renal amyloidosis.1  The dog illustrated here is not showing signs of renal compromise at the time of diagnosis (BUN and creatinine within normal ranges, absence of proteinuria); not all affected Shar Pei will develop renal disease. iv.  Shar Pei fever is an autosomal recessive trait. v.  Familial renal amyloidosis has been described in cats as well as dogs. In the cat, Abyssinian and Siamese are overrepresented. Dog breeds include Beagles, English foxhounds, and Walker foxhounds.

Note 1. Segev G, Cowgill LD, Jessen S, Berkowitz A, Mohr CF, Aroch I. Renal Amyloidosis in Dogs: A Retrospective Study of 91 Cases with Comparison of the Disease between Shar-Pei and Non-Shar-Pei Dogs. J Vet Intern Med . 2012;26(2):259– 268. doi:10.1111/j.1939-1676.2011.00878.x. 85  i. The sediment exam shows significant bacteriuria with moderate to severe pyuria and mild hematuria. The bacterial population is mixed, with predominantly rod-forming bacteria present. This is diagnostic for a severe bacterial urinary tract infection. ii. The most common organism cultured from the urinary tract infections in dogs is Escherichia coli , representing approximately 50% of cases. After E. coli , the next most common bacterial group identified in UTIs of dogs is the Gram-positive cocci: Staphylococcus , Streptococcus , and Enterococcus  spp., together these three genera account for around one-third of bacterial UTIs in dogs. A variety of bacterial genera, including Proteus , Klebsiella , Pseudomonas , and Mycoplasma  spp. account for the remaining cases of bacterial UTIs in dogs. iii.  The most commonly recognized comorbid conditions at presentation in dogs with diabetic ketoacidosis are, in descending order, pancreatitis, bacterial urinary tract infection, and hyperadrenocorticism.

138

86: Questions 86  A 9-year-old male intact Boxer dog presents with a history of sudden onset episodes of weakness and collapse. The owner reports that these episodes usually, but not always, occur after the dog has been exercising. On physical examination the dog is bright, alert, and responsive and body temperature and respiratory rate are within normal limits. The dog has marked bradycardia and his pulse is bounding. You do not perceive any pulse deficits. Figure  86.1 shows leads I, II, and III from a diagnostic ECG trace.

FIGURE 86.1 

i.  What rhythm abnormality do you detect? ii.  Given the dog’ s breed, what other rhythm abnormalities would you be concerned about? Is this ECG consistent with that disorder? iii.  Why does this dog have a bounding pulse? iv.  What treatment options and prognosis would you offer the dog’ s owner?

139

86: Answers 86  i.  The ECG shows a third-degree atrio-ventricular (AV) block. P waves are visible at an underlying rate of ~150 beats/minute (Figure  86.2, indicated by asterisks), these P waves are not accompanied by QRS complexes. There is a multifocal ventricular escape rhythm (Figure  86.2, indicated by V symbols) that is markedly slower than the atria, ~60  BPM.

FIGURE 86.2 

ii. In the Boxer dog, syncopal episodes should raise suspicion for arrhythmogenic right ventricular cardiomyopathy (ARVC). The ECG strip presented here is not consistent with this disorder for the time recorded. ARVC is a paroxysmal ventricular tachycardia rather than a bradycardia. Third-degree AV block would not typically be associated with ARVC, but a longer period of monitoring, potentially with a Holter monitor, would be necessary to completely rule out ARVC. iii.  Pulse character is determined by, among other factors, the ventricular stroke volume, the pressure differential between systolic and diastolic pressure, and the state of the peripheral vascular system. Ventricular filling is a major determinant of the stroke volume. When the patient has bradycardia, as seen here, the ventricular filling time is increased, which results in increased ventricular stroke volume. This is perceived as a pronounced increase in the pulse pressure. iv.  Unfortunately, treatment options are limited. The best option would be the insertion of a ventricular pacemaker to normalize the ventricular rhythm and improve cardiac output. Oral beta-agonist medications (albuterol, terbutaline) and methylxanthines (theophylline, aminophylline) can be trialed but usually are not effective. The prognosis for this dog is guarded to poor. Uncontrolled third-degree AV block is associated with significant risk for sudden death. The dog from which this trace was obtained showed acute decompensation and died within 12  hours of the trace being recorded.

140

87: Questions 87  A 7-year-old Bernese mountain dog presents to you with dyspnea and tachypnea and you diagnose a pleural effusion. The provided radiographs (Figure 87.1) show lateral (left and right views) and dorsoventral images of the thorax obtained after  thoracocentesis and pleural drainage.

FIGURE 87.1 

i.  How do you interpret the provided radiographs? What is your diagnosis? ii.  Are there any dog types or breeds that are predisposed to this condition? iii.  Which lung lobes are most commonly affected? iv.  How would you plan to treat this dog?

141

87: Answers 87  i.  In Figure 87.1, the left lateral (A), dorsoventral (B), and right lateral (C) survey radiographs show a focal, severe alveolar lung pattern throughout the entire left cranial lung lobe, including both subsegments. The lung lobe is enlarged, as suggested by caudally rounded borders (best seen on the DV radiograph). An air bronchogram is visible in the left cranial thorax. This bronchus also has an abnormal cranial-dorsal deviation (best seen in both lateral radiographs). The right lateral radiograph (C) shows marked focal narrowing (black arrow) of an abnormally positioned bronchus. There is axial rotation of the bifurcation of the trachea and main stem bronchi. Small indistinct gas bubbles are scattered throughout the region of the cranial subsegment of the left cranial lung lobe. There are small areas of residual pleural effusion, visible as increased soft tissue and widened pleural fissure lines, throughout the thorax. The heart is shifted toward the left of the midline. These radiographic findings are most consistent with a lung lobe torsion of the left cranial lung lobe, with an accompanying small volume of pleural effusion. The small gas attenuating areas (bubbles) throughout the left cranial lung lobe are concerning for left cranial lung lobe necrosis. Lung neoplasia is a possible differential but is considered less likely due to the abnormal position of the bronchus of the left cranial lung lobe. The gas bubbles distributed throughout the thorax could be secondary to the thoracocentesis or could be secondary to lung necrosis, leakage of air into the pleural space or formation of bullae. ii.  Lung lobe torsion is a relatively rare condition but is most common in larger-breed, deep-chested dogs. There is also some evidence that Pugs are predisposed. Lung lobe torsions have been reported in dogs with pleural effusion, such as due to chylous or inflammatory effusion as well as due to hemorrhagic effusion secondary to neoplasia such as hemangiosarcoma or post-severe thoracic trauma. iii.  The most commonly affected lung lobes are the right middle and left cranial, as seen here. iv.  Definitive medical management of lung lobe torsion is not feasible. Initial efforts should emphasize stabilization of the patient through drainage of the pleural effusion, provision of supplemental oxygen, and provision of pain control. If a preexisting condition is identified (such as bronchopneumonia), appropriate management plans should be made for the post-surgical period. The definitive treatment for lung lobe torsion is surgical resection of the affected lobe(s). Often the affected lobes are highly friable and necrotic; failure to resect the affected tissue can lead to pneumothorax and/or pyothorax.

142

88: Questions 88  A 4-year-old female spayed domestic shorthair cat is evaluated for persistent inappetence. The owner indicates the patient has been abnormally quiet and noninteractive for the past few days. She last ate 3 days ago and has shown minimal interest in food since that time. The family adopted a new kitten 3 weeks previously; the patient has acted aggressively toward the kitten since adoption. The kitten is reported to be healthy. Clinically, the cat appears very quiet on evaluation but is responsive. She weighs 7.5 kg (16.5 lb) with a BCS of 8/9. Her temperature, heart rate, and respiratory rate are within normal limits. Mucus membranes are slightly yellow but moist, with a CRT 2  μ g /dL, total T4  concentration within the reference interval, and a negative acetylcholine receptor antibody titre. Therefore a diagnosis of idiopathic megaesophagus was made. iv.  Because there was no evidence of aspiration pneumonia, antimicrobial therapy was not initiated. Patients should be fed and given water in an upright/vertical position using a Bailey chair (pictured to the right) and should be kept vertical for 15– 20  minutes after feeding. Some dogs best tolerate liquid diets, while other dogs best tolerate balls of food that are swallowed whole. When an adequate amount of food cannot be successfully administered by mouth, gastrostomy tube placement is advised. Cisapride and metoclopramide may reduce gastro-esophageal reflux. However, they have no effect on esophageal motility and their use in these patients is controversial. The long-term prognosis with idiopathic megaesophagus is guarded/poor, as patients often succumb to recurrent aspiration pneumonia. If the underlying disease can be treated successfully, the prognosis with secondary FIGURE 89.2  megaesophagus is fair/guarded.

150

90: Questions 90  You have been managing an 11-year-old male neutered Miniature Schnauzer for myxomatous mitral valve disease for a period of several months. The dog has been managed with a combination of enalapril and pimobendan and has maintained a good quality of life and activity levels. The dog presents with an acute onset of dyspnea, with increased respiratory effort. Lateral and dorsoventral radiographic views of the thorax are provided (Figure 90.1, lateral, and Figure 90.2, dorsoventral). i.  What is your most likely clinical diagnosis? ii.  How common is this complication in dogs with myxomatous mitral valve disease? iii.  Which valve leaflet is most commonly affected by ruptured chordae tendineae? iv.  How would you plan to treat this dog? What prognosis would you give?

FIGURE 90.1 

FIGURE 90.2 

151

90: Answers 90  i.  The combination of acute decompensation and congestive heart failure (evidenced by the onset of marked pulmonary edema) in a dog with prior evidence of myxomatous mitral valve disease (MMVD), is most suspicious for chordae tendineae rupture. Note that the left atrium is not dramatically enlarged in this dog, in the presence of significant pulmonary edema. This suggests that an acute left atrial volume overload is occurring, resulting in pulmonary edema, but the left atrium has not yet had an opportunity to dilate in response to the volume overload. The clinical diagnosis of ruptured chordae tendineae was subsequently confirmed in this dog via echocardiography. ii.  In a large retrospective study of dogs with MMVD incorporating >700 dogs, ruptured chordae tendineae occurred in 16% of cases.1 iii.  The most commonly affected chordae tendineae are the primary chordae of the anterior mitral valve leaflet. This results in this leaflet of the mitral valve inverting in to the left atrium during systole, creating a characteristic echocardiographic finding of a “flail leaflet.” iv.  The treatment of chordae tendineae rupture, in the acute stages, involves the use of intravenous diuretics (furosemide), oxygen supplementation, and strict cage rest. The use of vasodilators, such as nitroprusside or percutaneous glyceryl trinitrate, may be indicated in more severely affected dogs, but there is no strong data to suggest that use of venodilators is associated with better outcomes. Chordae tendineae rupture is considered a major complication of MMVD and has traditionally been given a poor prognosis, but in the large retrospective study previously cited, median survival overall was 425 days for dogs receiving appropriate stabilization treatment and successfully transitioning to maintenance diuretic therapy.

Note 1. Serres F, Chetboul V, Tissier R, et al. Chordae tendineae Rupture in Dogs with Degenerative Mitral Valve Disease: Prevalence, Survival, and Prognostic Factors (114 Cases, 2001–2006). J Vet Intern Med. 2007;21(2):258–264. doi:10.1111/j.1939-1676.2007.tb02958.x.

152

91 & 92: Questions 91  A 1-year-old spayed female Golden Retriever presents with a 1-week history of a productive cough, lethargy, mucopurulent nasal discharge, and hyporexia. She was adopted from an animal shelter 1 week prior to the development of clinical signs. Her vaccine history is unknown, but she was heartworm antigen-positive at the shelter. On physical examination, she is febrile (T:104°F), tachypneic (RR: 60), has bilateral mucopurulent nasal discharge, facial myoclonus, and hyperkeratosis of her foot pads and nasal planum. Harsh lung sounds diffusely and crackles in her right cranial lung field could be auscultated. No other abnormal findings were noted on physical examination. A left lateral thoracic radiograph is shown.

FIGURE 91.1 

i.  What is the most likely cause of the constellation of signs? ii.  What antemortem diagnostic tests could be used to confirm the diagnosis? iii.  The dog’  s vaccination history is unknown. Which antemortem diagnos- FIGURE 91.2  tic tests would help to confirm the diagnosis in light of the unknown vaccination history? iv.  What is your interpretation of the radiographic findings? v.  What is the prognosis for recovery from disease? 92  Figure 92.1 shows an ultrasonography image of a section of jejunum in a dog with chronic weight loss. There is diffuse thickening of the small intestine palpable on clinical examination. i.  What lesion(s) do you see in this image? ii.  What changes could you possibly observe on routine clinical chemistry in this patient? FIGURE 92.1 

153

91 & 92: Answers 91  i.  Canine distemper virus (CDV). ii.  Virus isolation, reverse transcriptase polymerase chain reaction (PCR) performed on urine, blood, cerebrospinal fluid, or neutralizing antibody testing. iii.  Virus isolation, real-time reverse transcriptase PCR. iv.  There is a focal alveolar pattern within the right cranial lung lobe compatible with possible pneumonia. Also present are diffuse bronchointerstitial changes that could be compatible with heartworm-induced pneumonitis, other infectious/inflammatory disease (e.g., distemper), or allergic pneumonitis. Finally, right caudal pulmonary arterial changes are compatible with early heartworm disease. v.  Recovery from CDV infection is dependent on the immune status of the dog and the strain of the virus. However, the presence of myoclonus and other neurologic signs suggest a grave prognosis. 92  i.  The mucosa of the jejunal loop (labeled M in Figure  92.2) shows a diffuse “ tiger striping”  pattern with radial regions of hyper- and hypoechogenicity. In the normal dog, the ultrasound appearance of the mucosa is much more homogenous and is hypoechoic relative to the lumen and submucosa (the thin band labeled SM in Figure  92.2). While not absolutely specific, this ultrasound finding is highly suspicious for the presence FIGURE 92.2  of a lymphatic drainage abnormality, which may be primary (i.e., congenital lymphangiectasia) or secondary to other inflammatory or neoplastic diseases affecting the GI tract or draining mesenteric lymphatics. ii.  The most likely finding on routine clinical chemistry in this patient is the presence of hypoalbuminemia, suggestive of a protein-losing enteropathy. Panhypoproteinemia (with loss of the globulin as well as albumin fractions) is possible and is typically associated with worse disease. However, not all patients with a protein-losing enteropathy will show hypoproteinemia and intestinal protein losses should be considered likely in this patient unless other lesions are found to account for the reported weight loss. Other clinical chemistry changes that are common in dogs with protein-losing enteropathies include hypocalcemia (due to albumin binding of calcium) and hypocholesterolemia (due to fat/cholesterol malabsorption).

154

93 & 94: Questions 93  A 15-month-old neutered male cross breed dog has presented with a history of several weeks of waxing and waning lethargy, variable appetite, and a “ stiff neck”  (Figure  93.1). On physical examination, you note cervical pain and stiffness, with no other neurological signs. The dog is mildly febrile. The remainder of the physical examination is unremarkable. Routine biochemistry examination shows a mild elevation in alkaline phosphatase activity (103  I U/L, FIGURE 93.1  Reference Range: 0– 80), with no other notable findings. Complete blood count shows a moderate neutrophilia (14.7  ×  109 /L, Reference Range: 3.6– 11.5). A cerebrospinal fluid sample is collected and shows a neutrophilic pleocytosis (150 cells/µ L , >80% neutrophils) and increased CSF protein (4.0  g /L; 40  mg/dL, Reference Range: 75% of cases are seen in dogs less than 1 year of age). The development of intussusception reflects an abnormality in gastrointestinal motility, typically an increased motility in at least one segment of the gastrointestinal tract. This increased motility can result from a variety of primary causes, including acute or chronic enteritis, parasitism, the presence of foreign bodies, and following prior intestinal surgery (such as for removal of a foreign body). Intussusception should be strongly suspected in younger dogs with development of vomiting and lethargy in the post-operative period following intestinal surgery or following recovery from viral or bacterial enteritis. In older animals, chronic enteritis, parasitism, and foreign bodies are still potential causes of intussusception, but intestinal masses/neoplasia also become more likely.

170

103: Questions

FIGURE 103.1 

103  A 5-year-old neutered male Oriental cat presents with a history of altered liver enzyme activity detected on pre-anesthetic blood work and chronic vomiting episodes that have increased in frequency to one to two times weekly in the last month. On physical examination, the cat is bright, alert, and responsive; rectal temperature was normal (37.6° C , 99.6° F), the cat weighed 4.9  kg (11 lb) and had a body condition score of 4/9. Pertinent clinical chemistry results are given below. SI Units Analyte

Result

Conventional Units

Ref Range

Result

Ref Range

BUN

6.5

(2.2–5.8)

µ mol/L

39

(13–35)

mg/dL

Glucose

14.7

(3.9–6.9)

mmol/L

268

(70–125)

mg/dL

mmol/L

249

(75–175)

mg/dL

g/L

3.3

(2.6–4)

Cholesterol

6.5

(2–4.6)

Albumin

33

(26–40)

Total bilirubin

8.6

(0–8.6)

µ mol/L

0.5

(0–0.5)

Alkaline phosphatase

92

(10–70)

U/L

92

(10–70)

U/L

GGT

11

(1–8)

U/L

11

(1–8)

U/L

ALT

806

(5–65)

U/L

806

(5–65)

U/L

g/dL mg/dL

i.  Interpret the provided clinical chemistry results. What differential diagnoses are you considering for this cat? ii.  What additional diagnostic testing is indicated? iii.  What are the two major types of cholangitis/cholangiohepatitis syndrome (CCHS) in cats?

171

103: Answers 103  i.  Mild elevation in BUN is likely pre-renal, although a USG should be checked for greater certainty. The mild elevation in blood glucose concentration is most likely due to stress hyperglycemia. There is marked elevation (>10× upper normal) in ALT activity, ALP and GGT activity are only mildly elevated and other liver “ functional markers”  (albumin, cholesterol, bilirubin) are essentially unremarkable. Differential diagnoses considered for these abnormalities include CCHS, “triaditis,”  hyperthyroidism, and toxin exposure. ii.  Given the common occurrence of pancreatitis and intestinal disease as comorbidities of cholangitis/cholangiohepatitis syndrome in cats, assessment of these organs is indicated. Measurement of serum cobalamin, folate and Spec-fPL concentrations may help establish these diagnoses. Abdominal ultrasonography should be considered, carefully assessing hepatic parenchyma, pancreas, and gastrointestinal tract. Measurement of serum total T4 should be done to rule out hyperthyroidism. Hepatic biopsy may be necessary if less invasive diagnostics are not fruitful. iii.  There are two major types of CCHS in cats, suppurative and lymphocytic/plasmacytic. Suppurative CCHS is usually associated with bacterial invasion of the common bile duct/biliary tree and requires antibiotic therapy, while lymphocytic/plasmacytic CCHS is thought to be an immune-mediated disorder and is commonly linked with idiopathic inflammatory disease of the pancreas and intestine in cats. Differentiation between these two disorders may require hepatic biopsy and culture or culture of bile obtained by ultrasound-guided cholecystocentesis. Generally, patients with suppurative CCHS are more likely to present febrile, dull, and lethargic and will have a shorter history of illness before presentation.

172

104: Questions 104  A 5-year-old female spayed German shepherd dog presents with an owner complaint of gradual onset lethargy, variable appetite, and occasional mild diarrhea. In the 24 hours before presentation, the dog has been more markedly lethargic and was unwilling to go out for a walk. On physical examination, you note mild bradycardia (80 BPM) and significant loss of muscle mass around the head (Figure 104.1). Measurement of serum electrolytes shows hyponatremia (Na+ 133 mEq/L, Reference Range: 139–153) and hyperkalemia (K+ 7.2 mEq/L, Reference Range: 3.5–5.6). i.  Name some other clinical chemistry or hematology findings that you might expect in this case. ii.  What additional diagnostic testing, if any, is indicated in this case? iii.  This dog’s Na:K is 18.5, where normal dogs are expected to have an Na:K >28– 40. What is the sensitivity and specificity of this finding for the diagnosis of hypoadrenocorticism? Which other diseases or abnormalities can result in a low Na:K? iv.  What is the mechanism that results in the hyponatremia and hyperkalemia in patients with hypoadrenocorticism? FIGURE 104.1 

173

104: Answers 104  i.  Hypoadrenocorticism is commonly, but not invariably, associated with a variety of other clinical chemistry and hematology findings. Common findings include hypercalcemia, azotemia, hypocholesterolemia, eosinophilia, and the absence of a stress leukogram in an obviously unwell patient. This dog exhibited azotemia with a normal range SDMA concentration and the absence of a stress leukogram at presentation. ii.  The gold standard test for the diagnosis of hypoadrenocorticism is an ACTH stimulation test. Neither of basal cortisol concentration or the presence of low Na:K are sufficiently sensitive or specific to confirm this diagnosis. A number of other diseases are associated with low Na:K; these are discussed in more detail below. This dog’s ACTH stimulation results were